Sei sulla pagina 1di 69

An 8-year-old boy is taken to a pediatrician because of behavioral changes, mild intellectual deterioration,

and
"laziness." Over the next several months the boy develops increasing clumsiness and periodic,
involuntary, jerky
movements every 3 to 6 seconds. Visual deterioration is apparent upon visual field testing, and optic
atrophy is
evident on funduscopic examination. Cerebrospinal fluid studies do not show significant pleocytosis,
but
oligoclonal bands of IgG are present on CSF electrophoresis. The electroencephalogram shows
periodic
discharges that are synchronous with the periods of myoclonus. Computed tomography (CT) of the
head shows
low-density white matter lesions and cerebral atrophy. At the age of 10, the boy dies. Prior infection
with which of
the following agents was probably related to the patient's condition?

A. Measles virus

B. Mumps virus

C. Papilloma virus

D. Poliovirus

E. Varicella

Explanation:

The correct answer is A. The rare disease illustrated is subacute sclerosing panencephalitis (SSPE),
which
typically presents as in the question stem. SSPE appears to be due to a combination of persistent,
possibly
abnormal measles virus and to autoimmune damage caused by antibodies directed against the virus.
Many
patients developing SSPE have had measles at 2 years of age or younger; there is typically a six-year
interval
between measles infection and symptom development. A small proportion of cases have followed
vaccination
with live measles virus. Unfortunately, no effective therapy has been developed, although some
experimental
work with drugs such as isoprinosine shows some promise.

Post-infectious encephalomyelitis, rather than SSPE, can follow mumps (choice B) or varicella
(choice E). Latent
infection with the varicella virus (choice E) causes shingles (herpes zoster).

Papilloma viruses (choice C) are associated with warts. They do not usually infect the brain.

Poliovirus (choice D) causes gastrointestinal disturbances, viremia, and paralysis.

A 57-year-old man presents with an episode of shaking chills the previous night. He has now developed
right-sided pleuritic chest pain, fever, sweats, malaise, purulent sputum, and mild hemoptysis. On
examination,
the patient is diaphoretic but alert, with right basilar rales. Chest X-ray films show a right lower lobe
infiltrate with
blunting of the right costophrenic angle. Why is this patient's sputum filled with pus?

A. Teichoic acids and peptidoglycan are chemotactic for neutrophils

B. The capsule of the causative agent is chemotactic for neutrophils

C. The causative agent is an intracellular organism

D. The causative agent is beta hemolytic

E. The organism produces an IgA protease

Explanation:

The correct answer is A. The answer to this question requires that the student realize that pus consists
of
bacteria and dead and dying neutrophils. This fact, taken along with the highly characteristic case
history,
reflects that the patient has a typical pneumonia. In the United States, the most common agent of this
would be
Streptococcus pneumoniae, a gram-positive extracellular pathogen rich in teichoic acids and
peptidoglycan,
which elicit the neutrophilic exudate.

The capsule of this organism is a polysaccharide and primarily elicits an antibody response rather
than
attracting neutrophils (choice B).

Streptococcus pneumoniae is an extracellular, not an intracellular (choice C) organism.

Streptococcus pneumoniae is alpha hemolytic, not beta hemolytic (choice D).

Streptococcus pneumoniae does produce an IgA protease (choice E) that enhances the ability of the
organism
to infect the respiratory mucosa, but this does not contribute to pus formation.

A 48-year-old waitress presents to a physician with malaise, loss of appetite, nausea, moderate fever, and
jaundice. Laboratory tests indicate a marked increase in serum transaminases. Serology for hepatitis
viruses is
performed and indicates positive results for the presence of HBsAg, HBc IgM antibody, and HCV
antibody.
Antibody tests for HBsAb and HAV are negative. The results indicate:

A. A dual infection of HBV and HAV

B. Chronic hepatitis A infection

C. Chronic hepatitis B infection


D. Hepatitis C infection

E. The presence of an acute HBV infection.

Explanation:

The correct answer is E. The presence of hepatitis B surface antigen (HBsAg) along with hepatitis B
core IgM
antibody (HBc IgM Ab), and the absence of hepatitis B surface antibody (HBsAb) indicates the
presence of the
early stages of an acute infection with Hepatitis B. The presence of antibody to Hepatitis C (HCV)
only indicates
exposure, but not a specific time of exposure; however, 85% of patients who are infected with HCV
develop
chronic infections, indicating that this patient has an 85% chance of having a dual infection with
HBV and HCV.
The acute or chronic HCV infection can be confirmed by PCR.

A dual infection of HBV and HAV (choice A) is not plausible since the IgM anti-HAV serology is
negative.

Hepatitis A does not cause chronic disease (choice B).

Chronic HBV infection (choice C) is unlikely because the patient has HBc IgM Ab, which is
characteristic of an
acute infection, rather than a chronic infection.

Hepatitis C infection (choice D) is not confirmed by these data because the presence of HCV Ab only
indicates
exposure to the virus, and not the state of infection. This could be caused by exposure at some earlier
time
(the elevated serum transaminases might be due to HBV infection). An active or chronic HCV
infection can only
be confirmed by PCR.

An 8-year-old girl is brought to the pediatrician for a severe sore throat. The pediatrician prescribes
penicillin and
sends the girl home. Later that day, she develops a diffuse maculopapular rash, shortness of breath,
and
wheezing. Her parents take her to the emergency department, where she is diagnosed with anaphylaxis
and
treated successfully with epinephrine. Three months later, the girl has dysuria and urinary frequency.
Urine
Gram's stain reveals numerous gram-negative rods. Which of the following antibiotics could be safely
given to
this patient?

A. Ampicillin

B. Aztreonam

C. Cefoperazone
D. Cephalexin

E. Methicillin

F. Oxacillin

G. Ticarcillin

Explanation:

The correct answer is B. Aztreonam is a monobactam. Since the basic ring structure is different from
penicillins,
there is no cross-allergenicity, and it can be safely given to those who have had severe reactions to
penicillins.
Aztreonam is highly active against gram-negative bacteria, but has no activity against gram-positive
bacteria or
obligate anaerobes. It is not β-lactamase resistant.

In general, there is cross-allergenicity between all the penicillins, since most people react to
breakdown
products of the β-lactam ring structure common to all penicillins. Patients who have experienced
an allergic
reaction to a penicillin may also be sensitive to cephalosporins.

Ampicillin (choice A) is a third-generation penicillin. It is a broad-spectrum penicillin that is active


against certain
gram-negative and gram-positive bacteria. It is also one of the main antibiotics (along with
clindamycin) that can
lead to antibiotic-induced pseudomembranous colitis.

Cefoperazone (choice C) is a third-generation cephalosporin. It is a broad-spectrum cephalosporin


that is active
against many gram-negative and gram-positive bacteria. It contains a methylthiotetrazole side chain
that can
cause a vitamin K deficiency and disulfiram-like reaction to alcohol. Many people also experience
diarrhea as a
side effect. It is a drug of choice in people with impaired renal function because 60% is eliminated by
the biliary
route.

Cephalexin (choice D) is a first-generation cephalosporin. It is mainly active against gram-positive


bacteria.

Methicillin (choice E) is a second-generation penicillin. It is active against many gram-positive


organisms. When
given in high doses for more than 2 weeks, it can cause interstitial nephritis, hepatitis, and
neutropenia.

Oxacillin (choice F) is a second-generation penicillin. It is used against many gram-positive


organisms. When
used in high dose for more than 2 weeks, it can cause hepatitis and neutropenia.

Ticarcillin (choice G) is a fourth-generation penicillin. Fourth-generation penicillins are extended-


spectrum
agents that are active against many gram-positive and gram-negative bacteria, including Pseudomonas
and
many Enterobacteriaceae. Ticarcillin is given with clavulanic acid, a β-lactamase inhibitor.
Since ticarcillin
is a disodium salt, it causes a large salt load, which can lead to salt retention and hypokalemia.

A 38-year-old woman with a history of multiple sexual partners is most at risk for which of the following?

A. Bladder carcinoma

B. Cervical carcinoma

C. Endometrial carcinoma

D. Ovarian carcinoma

E. Rectal carcinoma

Explanation:

The correct answer is B. In a sense, cervical carcinoma can be considered a "sexually transmitted
disease,"
since human papilloma virus (typically strains HPV-16 or HPV- 18) is transmitted venereally. In our
culture,
HPV-related dysplasia of the cervix is common, even in the 18-28 year old group, and carcinomas of
the cervix
(more rare now than in the past due to screening and aggressive therapy of dysplasia) can occur in the
35-45
age group. The other cancers listed typically occur in older age groups and are not as clearly related to
venereal agents.

A Brazilian immigrant is hired at a meat-packing plant, and undergoes an employment physical. Chest x-
ray
demonstrates a patchy, bilateral pneumonia and a lung mass, and he is referred to a specialist. Biopsy
of the
mass demonstrates fungal organisms with a few very distinctive "pilot's wheel" yeast forms. Which of
the following
is the most likely diagnosis?

A. Blastomycosis

B. Coccidioidomycosis

C. Histoplasmosis

D. Paracoccidioidomycosis

E. Sporotrichosis
Explanation:

The correct answer is D. Latin America and "pilot's wheel" budding yeast are clues for
paracoccidioidomycosis.
This disease is caused by Paracoccidioides brasiliensis, a dimorphic fungus that is found as a multiply-
budding
yeast in tissues.

Clues for questions about blastomycosis (choice A) would include spending time in states east of the
Mississippi
River and fairly large yeast.

Clues for questions about coccidioidomycosis (choice B) would be spending time in the southwestern
deserts of
the United States and spherules filled with endospores.

Clues for questions about histoplasmosis (choice C) would be spending time in the Ohio, Mississippi,
and
Missouri River Valleys and finding tiny yeast forms in macrophages.

Clues for questions about sporotrichosis (choice E) would be skin lesions in rose gardeners.

A psychotic, indigent man with a history of multisubstance abuse has been involuntarily hospitalized for 1
week.
Because of persistent diarrhea, stools are sent for ova and parasites, revealing numerous granular,
spherical,
thin-walled cysts measuring 10-20 µm in diameter. Trichrome stains show up to four nuclei in most of
the cysts.
These finding are consistent with an infection by which of the following organisms?

A. Cryptosporidium parvum

B. Dientamoeba fragilis

C. Entamoeba histolytica

D. Giardia lamblia

E. Isospora belli

Explanation:

The correct answer is C. Entamoeba are relatively common enteric pathogens that can produce
asymptomatic
infection or more severe disease characterized by mucosal ulcerations and submucosal spread causing
abdominal distress and liquid stools. Stools may show either trophozoite forms or the typical
spherical cysts.
Several species of Entamoeba are seen, including Entamoeba coli and E. hartmanni. E. histolytica
cysts
characteristically are spherical in shape, 10-20 µm in diameter, and have granular cytoplasm
containing 1, 2, or
4 nuclei.
Cryptosporidium parvum(choice A) infections occur in the immunocompromised population and may
cause
severe diarrhea. The organism presents as minute (2-5 µm) intracellular spheres or arc-shaped
merozoites
under normal mucosa, and can be difficult to appreciate by light microscopy. Cysts in the stool are
too small
(4-5 µm) to be confused with Entamoeba.

Dientamoeba fragilis(choice B) is an intestinal amoeba that also may produce an infectious diarrhea.
It does not
have a cyst form, and only the trophozoite forms are seen in stools.

Giardia lamblia(choice D) is a flagellate protozoan that infects the stomach and small intestine when
contaminated water is ingested. Diagnosis is usually made by examining duodenal contents; however,
the stools
may contain the oval or elliptical cysts, which are thick-walled and measure 8-14 µm in diameter.
Spherical cysts
are not seen in Giardia infection.

Isospora belli(choice E) produces self-limited intestinal infections mostly in the tropics, where fever
and diarrhea
may last weeks to months. The stool-borne cysts are much larger than Entamoeba (30 x 15 µm), are
asymetrical, and are typically almond-shaped.

An autopsy is performed on a man who suddenly began vomiting voluminous quantities of blood and
exsanguinated.
The man's stomach is presented in the accompanying photograph. Which of the following organisms is
most likely
implicated in the pathogenesis of this disease?

A. Cryptosporidium parvum

B. Entamoeba histolytica

C. Escherichia coli

D. Helicobacter pylori

E. Mycobacterium tuberculosis

Explanation:

The correct answer is D. The gross photograph shows a stomach with a perforated peptic ulcer.
Perforation of a
peptic ulcer is potentially fatal, because of either peritonitis with sepsis or sudden exsanguination (if the
perforation
damages one of the many arteries of the stomach). Peptic ulcer disease, gastritis, and possibly gastric
carcinoma
and gastric lymphoma have been strongly associated with Helicobacter pylori colonization of the mucus
layer
covering the gastric mucosa. Colonization is associated with destruction of the mucus layer, thereby
destroying its
protective function.

Cryptosporidium parvum(choice A) causes diarrhea that is severe in immunocompromised patients.

Entamoeba histolytica(choice B) produces dysentery-like symptoms or can cause liver abscess.

Escherichia coli(choice C) causes a variety of diarrheal diseases and can infect the bladder and soft
tissues.

Mycobacterium tuberculosis(choice E) causes tuberculosis, characterized by granuloma formation,


especially in the
lungs.

A 34-year-old HIV-positive man without previous opportunistic infections presents complaining of


dyspnea with
daily activity. He states that he has had a mild cough and fever but denies having had chills, sputum
production,
or chest discomfort. Physical examination is remarkable for oral thrush and a few small, nontender
cervical lymph
nodes. A chest x-ray film reveals bilateral interstitial infiltrates, and bronchoalveolar lavage reveals
small
silver-staining cysts. In which other patient population in the U.S. is this organism a frequent cause of
a
life-threatening pneumonia?
A. Bone marrow transplant patients

B. Hospitalized adults on antibiotic therapy

C. Late-term pregnant women

D. Normal adults in periods of stress

E. Premature infants

F. Preschool children

G. Sexually active adolescents

H. Third-trimester fetuses

Explanation:

The correct answer is E. This patient has Pneumocystis carinii pneumonia, the most common atypical
pneumonia in AIDS patients. The next most common pool of at-risk individuals consists of premature
infants.

Bone marrow transplant patients (choice A) would be immunologically compromised, and might be a
second
at-risk group (behind premature infants), but would not be the first choice on this list.

Hospitalized adults on antibiotic therapy (choice B) might be slightly immunologically compromised


or stressed,
but are not the major patient pool at risk for this infection in the U.S.

Late-term pregnant women (choice C) are not immunologically compromised and would not be at
risk.

Normal adults in periods of stress (choice D) are not particularly susceptible to opportunistic
pathogens.

Preschool children (choice F) have normal rates of exposure to P. carinii, as would any adult, but are
not
susceptible to serious pneumonia with this agent unless they are immunologically suppressed.

Sexually active adolescents (choice G) would have the same infection rates as normal infants and
adults, but
again would not be susceptible to life-threatening pneumonia. P. carinii is transmitted via aerosols,
not sexual
activity.

Third-trimester fetuses (choice H) are not susceptible to infection with this fungus, although they may
become
infected in utero with HIV.

A 16-year-old high school cheerleader presents with low grade fever, pleuritic pain and a non-productive
cough.
Her serum agglutinates Streptococcus salivarius strain MG. Therapy should include which of the
following?

A. Ampicillin

B. Erythromycin

C. Oxygen and external cooling

D. Penicillin G

E. Ribavirin

Explanation:

The correct answer is B. The patient has primary atypical pneumonia caused by Mycoplasma
pneumoniae.
These organisms are fastidious and difficult to culture in the laboratory, however serodiagnosis can be
most
helpful. Patients produce one or two heterophile antibodies during the course of the infection: one
agglutinates
human O+ RBCs in the cold (the cold hemagglutinins) while the other causes the agglutination of a
strain of
Streptococcus salivarius termed strain MG (the Strep MG agglutinins). Mycoplasma are susceptible
to the
macrolide family of antibiotics (erythromycin, clarithromycin, etc.). These organisms lack a cell wall,
hence they
are indifferent to antibiotics that interfere with peptidoglycan synthesis, such as penicillin (choice D)
and
ampicillin (choice A).

Oxygen and external cooling (choice C) are therapeutic measures that are used in the treatment of
severe
respiratory diseases such as pneumococcal pneumonia.

Ribavirin (choice E) is used in the treatment of respiratory syncytial virus infection in infants. This is
the most
common cause of hospitalization for respiratory disease in the very young, probably because aerosol
administration of the antiviral compound is best accomplished in a hospital setting.

A 24-year-old summer camp counselor complains of a severe headache and weakness. His condition
rapidly
deteriorates over a period of hours, and he is airlifted to a nearby hospital. A lumbar puncture is
performed and a
Gram's stain of spinal fluid reveals gram-negative diplococci. Infection with this organism is also
associated with
which of the following?

A. Dysentery

B. Erythema chronicum migrans


C. Myocarditis

D. Ophthalmia neonatorum

E. Waterhouse-Friderichsen syndrome

Explanation:

The correct answer is E. The Neisseria are gram-negative diplococci, and have two clinically
important species,
N. meningitidis and N. gonorrhoeae.N. meningitidis can either cause a sudden, fulminant, life-
threatening
meningitis or meningococcemia with a vasculitic purpura and disseminated intravascular coagulation.
The latter
presentation may be complicated by adrenal involvement, precipitating the usually fatal
Waterhouse-Friderichsen syndrome with coagulopathy, hypotension, adrenal cortical necrosis, and
sepsis. N.
gonorrhoeae causes gonorrhea and ophthalmia neonatorum (choice D), a neonatal eye infection.

Classical dysentery (multiple, small-volume stools with blood, mucus, and pus associated with
abdominal cramps
and tenesmus); (choice A) is caused by Shigella dysenteriae. A similar syndrome can be caused by
other
bacteria or amoebae as well.

Erythema chronicum migrans (choice B) is the pathognomonic dermatologic manifestation of Lyme


disease. This
skin lesion consists of an expanding erythematous lesion, with central clearing.

Bacterial myocarditis (choice C) is uncommon, and when it does occur, it usually involves
Staphylococcus aureus
or Corynebacterium diphtheriae.

A British dairy farmer develops fever with chills, myalgias, headache, skin rash, and vomiting. He is quite
ill and is
hospitalized. Blood cultures demonstrate tightly coiled, thin, flexible spirochetes shaped like a
Shepherd's crook.
The spirochetes are easily cultured in serum-enriched nutrient agar. Which of the following organisms
should be
suspected?

A. Brucella abortus

B. Brucella melitensis

C. Leptospira interrogans

D. Pseudomonas mallei

E. Pseudomonas pseudomallei
Explanation:

The correct answer is C. The only spirochete among the choices is Leptospira interrogans, so even if
you didn't
know the diseases these organisms produce, you may have been able to answer the question.
Leptospirosis,
which this patient has, is caused by a spirochete; if you were presented with a list of spirochetes in the
choices,
the phrase "Shepherd's crook" should tip you off to Leptospira. Clinically, leptospirosis may range
from nearly
asymptomatic, or at least indistinguishable from other minor flu-like illnesses, to a potentially fatal
form (Wal's
disease) with jaundice, bleeding, renal failure, and skeletal muscle necrosis. Spread is via contact with
blood or
urine from infected animals, notably rats. Leptospirosis is found worldwide, but its more severe forms
are most
likely to occur in the tropics.

Brucella abortus (choice A) is a gram-negative coccus and is one of the causes of brucellosis.

Brucella melitensis(choice B) is a gram-negative coccus and is one of the causes of brucellosis.

Pseudomonas mallei (choice D) and Pseudomonas pseudomallei(choice E) are small gram-negative


bacilli that
cause melioidosis.

A patient is referred to a neurologist because of ataxia. Neurological examination reveals a loss of


proprioception
and a wide-based, slapping gate. Magnetic resonance imaging reveals degeneration of the dorsal
columns and
dorsal roots of the spinal cord. Which of the following organisms is most likely to have caused this
pattern of
damage?

A. Haemophilus influenzae

B. Herpes simplex I

C. Neisseria gonorrhoeae

D. Neisseria meningitidis

E. Treponema pallidum

Explanation:

The correct answer is E. The findings described are those of tabes dorsalis, a form of tertiary syphilis
caused
by Treponema pallidum. Tabes dorsalis, and other forms of tertiary syphilis, are now uncommon in
this country,
possibly because the common use of antibiotics may "treat" many unsuspected cases of syphilis.
Haemophilus influenzae (choice A) and Neisseria meningitidis(choice D) can cause meningitis.

Neisseria gonorrhoeae(choice C) causes gonorrhea, which usually does not involve the CNS.

Herpes simplex I (choice B) can cause an encephalitis that typically involves the frontal and temporal
lobes.

A patient presents to a physician with jaundice. Physical examination reveals a nodular, enlarged liver. CT
of the
abdomen shows a cirrhotic liver with a large mass. CT-guided biopsy of the mass demonstrates a
malignant
tumor derived from hepatic parenchymal cells. Infection with which of the following viruses would
most likely be
directly related to the development of this tumor?

A. Epstein-Barr virus (EBV)

B. Hepatitis B virus (HBV)

C. Human herpesvirus type 8 (HHV 8)

D. Human papillomavirus (HPV)

E. Human T-lymphocyte virus (HTLV-1)

Explanation:

The correct answer is B. The tumor is hepatocellular carcinoma, which usually develops in the setting
of
cirrhosis due to a variety of damaging agents, including hepatitis B virus (HBV) infection, alcohol
use, and
hemochromatosis.

EBV (choice A) is associated with Burkitt's lymphoma and nasopharyngeal carcinoma.

HHV 8 (a member of the herpes family, choice C) is associated with Kaposi's sarcoma.

HPV (human papillomavirus, choice D) is associated with cervical, penile, and anal carcinoma.

HTLV-1 (human T-lymphocyte virus, choice E) is associated with adult T-cell leukemia.

A 58-year-old alcoholic man with multiple dental caries develops a pulmonary abscess and is treated with
antibiotics. Several days later, he develops nausea, vomiting, abdominal pain, and voluminous green
diarrhea.
Which of the following antibiotics is most likely responsible for this patient's symptoms?

A. Chloramphenicol
B. Clindamycin

C. Gentamicin

D. Metronidazole

E. Vancomycin

Explanation:

The correct answer is B. Any time you see the development of diarrhea in the same question stem as
the words
"treated with antibiotics," you should immediately think of pseudomembranous colitis.
Pseudomembranous
colitis is caused by Clostridium difficile and typically occurs as a result of treatment with clindamycin
or
ampicillin. You would confirm your suspicion by sending a stool culture to be tested for the presence
of the C.
difficile toxin.

Chloramphenicol's (choice A) most test-worthy side effect is aplastic anemia, not diarrhea. In
addition, you
might have been able to eliminate this choice simply because of the extremely low probability that
this patient
would receive this antibiotic in the USA.

Gentamicin's (choice C) key side effects include ototoxicity and nephrotoxicity.

Metronidazole (choice D) and vancomycin (choice E) do not cause pseudomembranous colitis; they
are used to
treat it.

A 38-year-old AIDS patient presents to the clinic complaining of nausea, occasional vomiting and "bumps"
on his
groin. On physical examination, multiple, nontender, pedunculated reddish purple nodules in the
inguinal and
perirectal areas are observed. The patient's liver is palpable 8 cm below the right costal margin.
Routine
laboratory tests are unremarkable except for an alanine aminotransferase level of 58 and alkaline
phosphatase
of 90. He denies any foreign travel, but has two pet cats. Which of the following is the most likely
cause of this
patient's infection?

A. Bartonella henselae

B. Human papillomavirus

C. Molluscum contagiosum virus

D. Rickettsia prowazekii
E. Treponema pallidum

Explanation:

The correct answer is A. Bacillary angiomatosis is a disease that occurs primarily in AIDS patients,
and is
indicative of a defect in cell-mediated immunity. It is caused by either Bartonella henselae or
Bartonella
quintana. The domestic cat is the reservoir for these organisms and they are usually transmitted to
humans via
a cat scratch or cat bite. Patients with this illness usually have multiple skin lesions and
extracutaneous
manifestations involving liver and bone. Diagnosis is usually based on characteristic histopathologic
findings
including plump "epithelioid" endothelial cells and mitotic figures. A macrolide, such as
erythromycin or
azithromycin, is the drug of choice for the infection.

Human papillomavirus (choice B) causes warts. Infection can present as a sessile wart or as
condyloma
acuminatum, which are fleshy soft growths that coalesce into large masses. When cellular immunity
is
depressed, as in AIDS, the condylomata acuminatum proliferate.

Molluscum contagiosum virus (choice C) is a pox virus that is spread by close person-to-person
contact.
Infection produces a firm nodule that often becomes umbilicated, and may resolve by discharging its
contents.
In AIDS, the lesions do not resolve, but enlarge and spread.

Rickettsia prowazekii(choice D) is the cause of epidemic typhus. It is spread by the human body
louse,
Pediculus humanis. Its reservoirs are humans and flying squirrels.

Treponema pallidum(choice E) is the spirochete that causes syphilis. The characteristic primary lesion
is a
chancre (a painless, indurated ulcer) at the site of inoculation.

A 7-year-old girl develops a fever, conjunctivitis, photophobia, and a cough. Her pediatrician notes white
spots on
a bright red background on the girl's buccal mucosa. Within days, a rash begins around the hairline,
then
spreads to the trunk and extremities. One week later, the child suddenly begins to convulse, and loses
consciousness. She is taken to the emergency room, where involuntary movements and pupillary
abnormalities
are noted. Which of the following would most likely be seen on CNS biopsy?

A. Demyelination of white matter of cerebral hemispheres with abnormal giant oligodendrocytes

B. Perivenous microglial encephalitis with demyelination

C. Phagocytosis of motor neurons in the spinal cord


D. Severe hemorrhagic and necrotizing encephalitis of the temporal lobe with eosinophilic
Cowdry type A
inclusion in neurons and glia

E. Small granulomas with central caseation in the meninges

Explanation:

The correct answer is B. The initial history given is classic for measles, with the appearance of
Koplik's spots
(white spots on the buccal mucosa) followed by a rash beginning along the neck and hairline and
spreading to
the trunk and extremities. The sequela this child is experiencing is post-infectious encephalomyelitis,
which can
follow either infection with measles, varicella, rubella, mumps, or influenza, or vaccination with
vaccinia vaccine
or rabies vaccine derived from nervous tissue. Treatment is supportive, with a mortality of 15 to 40%;
survivors
frequently have significant permanent neurologic deficits. The pathologic finding is perivenous
microglial
involvement with demyelination.

Choice A describes the findings of progressive multifocal leukoencephalopathy, a demyelinating


disease caused
by infection with JC virus, especially in immunocompromised individuals.

Choice C describes the findings of poliomyelitis, a paralytic disease affecting the ventral horn of the
spinal cord
and motor cortex, caused by an enterovirus (poliovirus).

Choice D describes the findings in herpes encephalitis, which typically affects the inferomedial
temporal lobes
and orbitofrontal gyri.

Choice E describes the findings in tuberculous meningitis, caused by M. tuberculosis.

A poor African community is experiencing an epidemic of severe hepatitis. The mortality among pregnant
women
is particularly high. Which of the following viruses is the most likely cause of the epidemic?

A. Cytomegalovirus (CMV)

B. Hepatitis A virus (HAV)

C. Hepatitis C virus (HCV)

D. Herpes simplex I

E. Hepatitis E virus (HEV)

Explanation:
The correct answer is E. Hepatitis E is an important, and until recently, unrecognized cause of
epidemics of
enterically transmitted acute hepatitis. Hepatitis E is caused by an enterically transmitted virus that
occurs
primarily in India, Asia, Africa, and Central America. Infection with the virus is associated with a
very high
mortality among pregnant women.

CMV (choice A) can cause acute hepatitis, but the disease is usually mild and often goes
unrecognized, except
in profoundly immunosuppressed patients.

Hepatitis A virus (choice B) is the major cause of epidemics of enterically transmitted viral hepatitis,
but is not a
significant cause of mortality in pregnant women.

Hepatitis C virus (choice C) is usually transmitted parenterally, rather than enterically, and is not a
significant
cause of mortality in pregnant women.

Herpes simplex (choice D) usually causes significant hepatitis only in profoundly immunosuppressed
patients.

A medical worker has a needle-stick accident involving an empty syringe that had been previously used on
a
patient with a known hepatitis B infection. Which of the following is the most probable outcome for
the medical
worker?

A. Acute hepatitis followed by recovery

B. "Healthy" carrier

C. Persistent infection followed by recovery

D. Persistent infection progressing to chronic hepatitis

E. Subclinical disease followed by recovery

Explanation:

The correct answer is E. Hepatitis B infection can produce a wide variety of clinical outcomes. The
most
common outcome (60% to 65%), however, turns out to be subclinical disease followed by complete
recovery.
The other choices listed show other possible outcomes, and their statistical impact is considered with
the
discussions of individual choices.

Approximately 20% to 25% of infected persons develop acute hepatitis (choice A), which is followed
in 99% of
these cases by recovery and in about 1% of cases by fulminant hepatitis.

Approximately 5% to 10% of cases become "healthy" carriers (choice B).

Approximately 4% of cases develop persistent infection, 67% to 90% of which then recover (choice
C) and 10%
to 33% of which have chronic hepatitis (choice D).

A 2-day-old baby girl suddenly develops abdominal distention, progressive pallid cyanosis, and irregular
respirations. The newborn also has "refused" to breast-feed for the past 18 hours. If the mother was
treated for a
serious infection with antibiotics for 14 days up to and including the day of delivery, which of the
following
medications did the mother most likely receive?

A. Aztreonam

B. Chloramphenicol

C. Clindamycin

D. Metronidazole

E. Sulfamethoxazole/trimethoprim

Explanation:

The correct answer is B. Gray "baby" syndrome is a disorder that occurs in newborns who have either
received
chloramphenicol immediately after birth or whose mothers have received the medication close to the
delivery
date. Symptoms typically appear in the following order: abdominal distention with or without emesis,
progressive
pallid cyanosis, and vasomotor collapse, frequently accompanied by irregular respiration. Death can
occur as
early as a few hours after onset of signs and symptoms. Other symptoms may include: loose, greenish
stools, a
refusal to suck, ashen color (implied by the name gray baby syndrome), and lactic acidosis.
Chloramphenicol is
an antimicrobial agent used in the treatment of serious infections when less toxic alternatives are
inappropriate.

Aztreonam (choice A) is a beta-lactam antibiotic used primarily in the treatment of gram-negative


infections of the
urinary tract, lower respiratory tract, and skin, and for intra-abdominal infections. The use of this agent
in
pregnant or nursing women and infants is considered to be safe and effective.

Clindamycin (choice C) is an anti-infective agent used in the treatment of serious infections when less
toxic
alternatives are inappropriate. Although the agent is considered to be safe and effective during
pregnancy, it is
associated with the development of pseudomembranous colitis and agranulocytosis.

Metronidazole (choice D) is an antibiotic used primarily in the treatment of anaerobic infections. The
use of
metronidazole should be restricted in pregnancy since newborns have a decreased ability to metabolize
this
medication. When the elimination of metronidazole is decreased, the severity of adverse reactions
increases.
Adverse reactions include peripheral neuropathy, seizures, irritability, and profound gastrointestinal
disturbances.

Sulfamethoxazole/trimethoprim (choice E) is associated with the development of kernicterus, which is


a disorder
that can cause abnormal cerebral development in infants. The majority of infants with this disorder
generally die
within a few weeks of birth. Those infants who survive are often mentally retarded, deaf, or physically
impaired.

A 65-year-old man presents with fever, severe headache, and nuchal rigidity. Physical examination in the
emergency department shows a Glasgow coma score of 7. Lumbar puncture reveals cloudy
cerebrospinal fluid
(CSF) with 1200 neutrophils/mm3, elevated protein, and decreased glucose. Which of the following is
the most
probable etiologic agent of this condition?

A. Arbovirus

B. Herpesvirus

C. Mycobacterium tuberculosis

D. Neisseria meningitidis

E. Streptococcus pneumoniae

Explanation:

The correct answer is E. The clinical manifestations (fever, headache, nuchal rigidity, and low
Glasgow coma
score), along with the CSF findings (increased neutrophils, elevated protein, and reduced glucose),
strongly
indicate acute pyogenic (bacterial) meningitis as the underlying condition. Of the microorganisms
listed, either
Neisseria meningitidis or Streptococcus pneumoniae can cause this form of meningitis; however,
Streptococcus
pneumoniae is by far the most frequent organism causing acute meningitis in elderly patients.

Arboviruses and herpesviruses (choices A and B) can cause an encephalitis characterized by


lymphocytic
infiltration of the brain parenchyma and leptomeninges. In this case, CSF findings would include an
increased
number of lymphocytes and a normal glucose concentration.

Mycobacterium tuberculosis(choice C) may cause a chronic meningoencephalitis, with a prolonged


clinical
course. It is characterized pathologically by a dense granulomatous infiltrate of the base of the brain.
Associated CSF findings include increased lymphocytes and normal or slightly decreased glucose.

Neisseria meningitidis(choice D) is the classic etiologic agent associated with acute pyogenic
meningitis, but it
usually affects adolescents and young adults. In neonates, the most frequent organisms include
Escherichia
coli and group B streptococci; in infants and children, Hemophilus influenzae; and in the elderly,
Streptococcus
pneumoniae and Listeria monocytogenes.

A 12-year-old boy has a productive cough characterized by large volumes of foul-smelling sputum. Three
years
ago, the patient was diagnosed with pancreatic insufficiency, as evidenced by repetitive
gastrointestinal symptoms
of steatorrhea. After culture of the sputum, colorless, oxidase-positive colonies with a fruity aroma
develop on the
agar. The function of which of the following proteins is most likely inhibited by the bacteria
responsible for this
boy's infection?

A. A CFTR protein lacking a phenylalanine in exon 10 at position 508

B. A GTP-binding protein involved in the elongation step of protein synthesis

C. A GTP-binding protein similar to the one coupled with α2-adrenergic receptors

D. A GTP-binding protein similar to the one coupled with β-adrenergic receptors

E. A phosphorylation-regulated chloride channel in the apical membrane of epithelial cells

Explanation:

The correct answer is B. The boy is displaying the characteristic symptoms of cystic fibrosis with
bronchiectasis
(accounting for the foul-smelling sputum) and pancreatic insufficiency, producing steatorrhea.
However, the
question directly relates to the infectious agent causing the pneumonia, Pseudomonas aeruginosa, an
oxidase-positive bacteria with a fruity aroma. P. aeruginosa is a strictly aerobic gram-negative rod that
produces
an exotoxin (exotoxin A) that ADP-ribosylates, and therefore inhibits, eukaryotic elongation factor 2
(eEF-2),
which is a G-protein involved in the eukaryotic translation of proteins.

The cystic fibrosis transmembrane conduction regulator (CFTR) protein (choice A) is the product of
the cystic
fibrosis gene, a large 24 exon gene located on the long arm of chromosome 7 (7q31). This protein has
two
nucleotide binding domains. In 70% of the families with the disease, a three base pair deletion at
codon 508 of
exon 10 results in the deletion of a phenylalanine (DF508). This codon is critical for one of the
nucleotide
binding sites of the CF gene product, resulting in poor function of a phosphorylation-regulated
chloride ion
channel in the apical membrane of epithelial cells. The defective chloride channel leads to impaired
secretion of
chloride in the lumen and promotes the absorption of sodium inside the cells. Water follows,
concentrating the
airway secretions. The increased viscosity of the airway secretions predisposes the patient to repeated
infections.

A GTP-binding protein similar to the one coupled with α2-adrenergic receptors (choice C)
refers to a
G-protein that inhibits adenylate cyclase, lowering cAMP. This G-protein is sensitive to Pertussis
toxin.

A GTP-binding protein similar to the one coupled with β-adrenergic receptors (choice D) refers
to a
G-protein that stimulates adenylate cyclase, increasing cAMP. This G-protein is sensitive to cholera
toxin.

The product of the cystic fibrosis gene is a phosphorylation-regulated chloride channel in the apical
membrane
of epithelial cells (choice E).

During the asymptomatic latent phase of AIDS, the virus is actively proliferating, and can be found in
association
with

A. B lymphocytes

B. follicular dendritic cells in lymph nodes

C. ganglion cells

D. oligodendrocytes

E. peripheral nerves

Explanation:

The correct answer is B. Follicular dendritic cells in the germinal centers of lymph nodes are
important
reservoirs of HIV. Although some follicular dendritic cells are infected with HIV, most viral particles
are found on
the surface of their dendritic processes. Follicular dendritic cells have receptors to the Fc portion of
immunoglobulins that serve to trap HIV virions coated with anti-HIV antibodies. These coated HIV
particles retain
the ability to infect CD4+ T cells as they traverse the dendritic cells.

B lymphocytes (choice A) have a surface marker (CD21 protein-a complement receptor) to which an
Epstein-Barr envelope glycoprotein can bind. The virus associates with the host cell genome,
producing a latent
infection. These B cells undergo polyclonal activation and proliferation.

Ganglion cells (choice C), particularly the satellite cells around the ganglion cells in the dorsal root
ganglia, can
be infected by varicella-zoster. Herpes type I and II infect neurons that innervate skin and mucous
membranes.

Oligodendrocytes (choice D) are directly infected by two viruses, JC virus (a polyomavirus) and
measles virus.
JC virus causes progressive multifocal leukoencephalopathy (PML), and measles virus produces a
latent
syndrome called subacute sclerosing panencephalitis (SSPE).

Peripheral nerves (choice E) are indirectly affected by HIV virus in the AIDS-associated myopathy.
The disease
is characterized by a subacute onset of proximal muscle weakness, sometimes with pain, and elevated
levels of
creatine kinase. The muscles and nerves are infiltrated with mononuclear cells, including HIV-
positive
macrophages.

A viral organism was isolated from a painful blister on the lip of a teenage girl. The agent was found to
double-stranded, linear DNA and was enveloped. The patient had a similar sore approximately 2
months ago.
Which of the following is the most likely causative organism?

A. Adenovirus

B. Coxsackie virus

C. Herpes simplex type 1 virus

D. Herpes zoster virus

E. Papilloma virus

Explanation:

The correct answer is C. Herpes simplex is an enveloped, linear DNA virus that is a very common
infectious
agent; most adults will have anti-Herpes simplex antibodies in their serum, although many may not
have ever
had any clinical signs of disease. The hallmark of this disease is painful skin vesicles often called
"cold" sores,
or "fever" blisters to denote the precipitating event that preceded the appearance of the lesions. The
virus has
a propensity to become latent in the host, finding safe refuge in nervous tissue. Activation of the
infection
occurs following mild trauma (e.g., a visit to the dentist), hormonal changes (e.g., menses), and
immunosuppression (e.g., following organ transplantation). Other, more serious, manifestations of
disease
include encephalitis, pneumonia, and hepatitis; these are particularly likely to be seen in
immunodeficient
patients such as those with AIDS.

Adenoviruses (choice A) are naked, linear, double-stranded DNA viruses that cause acute, usually
self-limiting,
influenza-like illnesses occurring in the fall and winter. The symptoms include pharyngitis, fever,
cough, and
general malaise. Epidemic pharyngoconjunctivitis and pneumonia can occur in closed populations
such as
military installations.

Coxsackie viruses (choice B) are naked, single-stranded, polycistronic viruses with an RNA genome.
They are
divided into groups A and B based on their virulence in suckling mice. Coxsackie A group causes
generalized
myositis and flaccid paralysis, which is rapidly fatal to infant mice, whereas group B produces less
severe
lesions of the heart, pancreas, and central nervous system. In man, Coxsackie A causes herpangina
and
hand-foot-and-mouth disease, while Coxsackie B is seen in patients with pleurodynia, myocarditis,
and
pericarditis. Both groups cause upper respiratory infections, febrile rashes, and meningitis.

Herpes zoster (choice D), the varicella virus, is an enveloped, double-stranded DNA virus that is a
very common
infectious agent in children. Chickenpox is a mild, self-limiting illness in children that is evidenced as
a fever
followed by a macular rash that progresses to papules, then vesicles of the skin and mucous
membranes.
Shingles is a recurrence of a previously latent varicella infection in which the virus has taken refuge
in sensory
ganglia of spinal or cranial nerves. Various factors that decrease the immune status of the patient
contribute to
the exacerbation of the infection. Severe dermatomal pain occurs with a vesicular eruption, fever, and
malaise.

Papilloma viruses (choice E) are members of the Papovavirus family. They are non-enveloped and
possess a
double-stranded, circular DNA genome. They cause skin, plantar, and genital warts; some serotypes
of human
papilloma viruses (e.g., HPV-16) are associated with penile, laryngeal, and cervical carcinomas.

A viral organism was isolated from a painful blister on the lip of a teenage girl. The agent was found to
double-stranded, linear DNA and was enveloped. The patient had a similar sore approximately 2
months ago.
Which of the following is the most likely causative organism?

A. Adenovirus
B. Coxsackie virus

C. Herpes simplex type 1 virus

D. Herpes zoster virus

E. Papilloma virus

Explanation:

The correct answer is C. Herpes simplex is an enveloped, linear DNA virus that is a very common
infectious
agent; most adults will have anti-Herpes simplex antibodies in their serum, although many may not
have ever
had any clinical signs of disease. The hallmark of this disease is painful skin vesicles often called
"cold" sores,
or "fever" blisters to denote the precipitating event that preceded the appearance of the lesions. The
virus has
a propensity to become latent in the host, finding safe refuge in nervous tissue. Activation of the
infection
occurs following mild trauma (e.g., a visit to the dentist), hormonal changes (e.g., menses), and
immunosuppression (e.g., following organ transplantation). Other, more serious, manifestations of
disease
include encephalitis, pneumonia, and hepatitis; these are particularly likely to be seen in
immunodeficient
patients such as those with AIDS.

Adenoviruses (choice A) are naked, linear, double-stranded DNA viruses that cause acute, usually
self-limiting,
influenza-like illnesses occurring in the fall and winter. The symptoms include pharyngitis, fever,
cough, and
general malaise. Epidemic pharyngoconjunctivitis and pneumonia can occur in closed populations
such as
military installations.

Coxsackie viruses (choice B) are naked, single-stranded, polycistronic viruses with an RNA genome.
They are
divided into groups A and B based on their virulence in suckling mice. Coxsackie A group causes
generalized
myositis and flaccid paralysis, which is rapidly fatal to infant mice, whereas group B produces less
severe
lesions of the heart, pancreas, and central nervous system. In man, Coxsackie A causes herpangina
and
hand-foot-and-mouth disease, while Coxsackie B is seen in patients with pleurodynia, myocarditis,
and
pericarditis. Both groups cause upper respiratory infections, febrile rashes, and meningitis.

Herpes zoster (choice D), the varicella virus, is an enveloped, double-stranded DNA virus that is a
very common
infectious agent in children. Chickenpox is a mild, self-limiting illness in children that is evidenced as
a fever
followed by a macular rash that progresses to papules, then vesicles of the skin and mucous
membranes.
Shingles is a recurrence of a previously latent varicella infection in which the virus has taken refuge
in sensory
ganglia of spinal or cranial nerves. Various factors that decrease the immune status of the patient
contribute to
the exacerbation of the infection. Severe dermatomal pain occurs with a vesicular eruption, fever, and
malaise.

Papilloma viruses (choice E) are members of the Papovavirus family. They are non-enveloped and
possess a
double-stranded, circular DNA genome. They cause skin, plantar, and genital warts; some serotypes
of human
papilloma viruses (e.g., HPV-16) are associated with penile, laryngeal, and cervical carcinomas.

Which of the following is a feature of gram-positive bacteria rather than gram-negative bacteria?

A. Lipid A-containing lipopolysaccharide

B. Lipoprotein in periplasmic space

C. Outer membrane

D. Peptidoglycan in periplasmic space

E. Thick peptidoglycan cell wall

Explanation:

The correct answer is E. Most of the features listed are those of gram-negative bacteria, which have a
complex
cell envelope consisting of a cytoplasmic (inner) membrane, a periplasmic space containing
peptidoglycan
(choice D) and lipoprotein (choice B), an outer membrane (choice C), and sometimes a capsule. The
outer
membrane contains lipopolysaccharide (choice A) which is a major component of endotoxin. The
peptidoglycan
cell wall of the gram-negative bacteria is thin, while that of the gram-positive bacteria is thick. Other
features of
gram- positive bacteria include a fairly simple surface with cytoplasmic membrane, peptidoglycan,
cell wall, and
sometimes, an outer capsule. The cell wall contains lipoteichoic acids.

A mailman gets a severe bite wound from a pit bull guarding a junkyard. The wound is cleansed and he
receives
a booster injection of tetanus toxoid and an injection of penicillin G. Several days later, the wound is
inflamed and
purulent. The exudate is cultured on blood agar and yields gram-negative rods. Antibiotic sensitivity
tests are
pending. The most likely agent to be isolated is

A. Bartonella henselae
B. Brucella canis

C. Clostridium tetani

D. Pasteurella multocida

E. Toxocara canis

Explanation:

The correct answer is D. Pasteurella multocida is a gram-negative rod that is normal flora of the oral
cavity of
dogs and cats. It often causes a local abscess following introduction under the skin by an animal bite.
Most
cases occur in children who are injured while playing with a pet.

Bartonella henselae (choice A) is a very small, gram-negative bacterium that is closely related to the
rickettsia,
although it is able to grow on lifeless media. It is the cause of cat-scratch disease (a local, chronic
lymphadenitis
most commonly seen in children) and bacillary angiomatosis (seen particularly in AIDS patients). In
this latter
patient population, the organism causes proliferation of blood and lymphatic vessels causing a
characteristic
"mulberry" lesion in the skin and subcutaneous tissues of the afflicted individual.

Brucella canis(choice B) is a gram-negative rod that is a zoonotic agent. Its normal host is the dog,
but when it
gains access to humans, it causes an undulating febrile disease with malaise, lymphadenopathy and
hepatosplenomegaly. The normal route of exposure is via ingestion of the organism.

Clostridium tetani(choice C) is a gram-positive spore-forming anaerobic rod. It causes tetanus (a


spastic
paralysis caused by tetanospasmin, which blocks the release of the inhibitory neurotransmitters
glycine and
gamma-aminobutyric acid [GABA]). There may be no lesion at the site of inoculation, and exudation
would be
extremely rare.

Toxocara canis (choice E), a common intestinal parasite of dogs, is a metazoan parasite that causes
visceral
larva migrans. Young children are most likely to be affected, as they are most likely to ingest soil
contaminated
with eggs of the parasite.

The World Health Organization identifies an alarming increase in hospital admissions worldwide
attributable to a
new and unexpected serotype of influenza A virus. The biological attribute of influenza A virus,
which allows the
sudden appearance of dramatically new genetic variants, is also present in a limited number of other
viral
families. Which of the following viruses also possesses this biological attribute?
A. Coronavirus

B. HIV

C. Measles virus

D. Rabies virus

E. Rotavirus

F. Rubella virus

G. St. Louis encephalitis virus

Explanation:

The correct answer is E. This case describes a pandemic of influenza A, which is caused by the ability
of the
virus to undergo dramatic genetic changes of type by reassortment of its segmented RNA genome - a
trait
called genetic shift. The only virus on the list that possesses a segmented genome is the rotavirus, in
the
reovirus family, which possesses 10-11 segments in its genome.

Coronavirus (choice A) is not segmented and is a cause of the common cold.

HIV (choice B) is not segmented and is known for its genetic drift (minor mutational changes over
time due to
an error-prone polymerase), not genetic shift.

Measles virus (choice C) is not segmented and is controlled largely by vaccination.

Rabies virus (choice D) is not segmented.

Rubella virus (choice F) is not segmented.

St. Louis encephalitis virus (choice G) is a flavivirus and is not segmented.

A 35-year-old woman presents to her gynecologist because of vaginal itchiness and discharge. Pelvic
examination demonstrates abundant white, curdy material in the vagina. Microscopic examination of
the material
demonstrates fungal hyphae and yeast forms. Which of the following systemic diseases can
predispose for this
condition?

A. Crohn's disease

B. Diabetes mellitus
C. Disseminated gonococcal infection

D. Rheumatoid arthritis

E. Systemic lupus erythematosus

Explanation:

The correct answer is B. The patient has vulvovaginitis secondary to Candida infection. Predisposing
factors
include a high vaginal pH, diabetes, and use of antibiotics. The increased vulnerability in diabetes
may reflect
increased glucose concentrations in vaginal secretions and relative immunosuppression.

Crohn's disease (choice A) can predispose for fistulas involving the vagina, rather than vulvovaginitis.

Dissemination of Neisseria gonorrhoeae(choice C) can cause septic arthritis.

Neither rheumatoid arthritis (choice D) nor systemic lupus erythematosus (choice E) are specifically
associated
with Candida vulvovaginitis.

A 1-week-old female infant with symptoms of vomiting and anorexia has a temperature of 102° F. A
bulging
fontanel is noted on physical examination. The most likely agent is

A. Haemophilus influenzae type b

B. Listeria monocytogenes

C. Neisseria meningitidis

D. Staphylococcus aureus

E. Streptococcus agalactiae

Explanation:

The correct answer is E. Vomiting, anorexia, high fever (above 100.4° F), and a bulging fontanel
equals
neonatal meningitis until proven otherwise. Streptococcus agalactiae (group B strep) and Escherichia
coli (not
an answer choice) are the most common causes in neonates up to 1 month of age. The next most
reasonable
response would have been Listeria monocytogenes(choice B), another, though less common, cause of
neonatal
meningitis.

Most cases of meningitis caused by Haemophilus influenzae(choice A) occur in children between 6


months and
6 years of age, 90% of which result from the capsular type b strain. It has become much less prevalent
since the
H. influenzae type b conjugate vaccine has been routinely administered to infants.

Neisseria meningitidis(choice C) is the most common cause of epidemic meningitis. The two
organisms most
often associated with sporadic cases are Haemophilus influenzae and Streptococcus pneumoniae (the
most
common cause in adults over 30).

Staphylococcus aureus(choice D) is not a common cause of meningitis, except in patients with CSF
shunts. It is
often responsible for abscesses, osteomyelitis, endocarditis, toxic shock syndrome, and food
poisoning.

A 7-month-old child presents with a 4-day history of fever, deepening cough, and dyspnea. A chest x-ray
shows
multiple interstitial infiltrates and hyperinflation of the lungs. Multinucleated giant cells with
cytoplasmic inclusion
bodies are seen when a nasal wash is inoculated into culture. The most appropriate therapy includes
administration of which of the following drugs?

A. Acyclovir

B. Ganciclovir

C. Ribavirin

D. Trifluorothymidine

E. Zidovudine

Explanation:

The correct answer is C. Ribavirin is an antiviral drug approved for the treatment of severe respiratory
syncytial
virus infection, the most common cause of pneumonia and bronchiolitis in children under 1 year of
age. It
should be given by aerosol.

Acyclovir (choice A) is a guanosine analog that is useful for the treatment of primary and recurrent
herpes
infections and herpes simplex virus encephalitis.

Ganciclovir (choice B) is a guanosine analog used in the treatment of cytomegalovirus retinitis and
cytomegalovirus infections in AIDS patients.

Trifluorothymidine (choice D) is a thymidine analog used topically for the treatment of recurrent
epithelial
keratitis and primary keratoconjunctivitis due to herpes simplex viruses.
Zidovudine (choice E) is a thymidine analog that inhibits reverse transcriptase. It is active against
human
retroviruses, including HIV-1, HIV-2, and HTLV-1.

A 6-year-old boy presents to the pediatric clinic with fever and earache. He has just finished an
unsuccessful
course of amoxicillin. On physical exam, his right tympanic membrane appears injected. Which of the
following
antimicrobials would be most appropriate to prescribe for this patient?

A. Amphotericin B

B. Bacitracin

C. Cefaclor

D. Erythromycin

E. Sulfamethoxazole

Explanation:

The correct answer is C. The drug of choice for otitis media in children is amoxicillin. But in
refractory cases,
often due to bacterial resistance, switching to a different drug class is often effective. You must look
for another
medication that is effective against common organisms responsible for pediatric otitis media, such as
Streptococcus pneumoniae (a gram-positive diplococcus) and Haemophilus influenzae (a gram-
negative rod). A
second-generation cephalosporin, such as cefaclor, should cover both and is the best choice.
Consequently, it
is commonly used in cases of amoxicillin-resistant otitis media. None of the other choices cover the
proper
spectrum of organisms.

Amphotericin B (choice A) is an antifungal polyene. It works by binding to ergosterol in the fungal


cell
membrane, creating an artificial pore. It is used to treat systemic mycoses such as Aspergillus,
Blastomyces,
Candida, Coccidioides, Cryptococcus, and Histoplasma.

Bacitracin (choice B) is a topical agent used to fight infection with gram-positive organisms. It
interferes with cell
wall synthesis.

Erythromycin (choice D) is a macrolide antibiotic that binds to the 23s rRNA portion of the 50s
subunit of
ribosomes, inhibiting release of uncharged tRNA and stopping protein synthesis. Though effective
against S.
pneumoniae, it is not particularly active against H. influenzae. Note that erythromycin may be used in
amoxicillin-resistant otitis media, but only when administered with a sulfonamide such as
sulfisoxazole.
Sulfamethoxazole (choice E) is a sulfonamide. It is bacteriostatic and works by inhibiting folic acid
synthesis. It
resembles p-aminobenzoic acid (PABA) structurally. When combined with trimethoprim (a
dihydrofolate
reductase inhibitor) it exerts a bactericidal effect and serves as the drug combination of choice for
complicated
urinary tract infections.

A previously healthy 18-month-old girl is brought to the office with 2 days of irritability, poor appetite, and
pulling
at her left ear. She has no known allergies, and her vaccinations are up-to-date. On examination, the
child's
temperature is 102.8 F. She is easily consoled by the mother and moves her neck spontaneously
without
discomfort. There is a clear discharge from the nares. The left tympanic membrane is erythematous,
dull, and
bulging. Which of the following virulence factors is generally absent in the strains of the causative
organism that
produce otitis media, compared with those that produce epiglottitis or meningitis?

A. β-Lactamase

B. IgA protease

C. Lipopolysaccharide

D. Nonpilus adhesins

E. Pili

F. Pneumolysin

G. Polyribitol phosphate

Explanation:

The correct answer is G. This is most likely a case of Haemophilus influenzae otitis media. 95% of all
cases of
invasive disease (epiglottitis, meningitis) due to H. influenzae are caused by type b organisms that
possess a
polyribitol phosphate capsule. Otitis media is generally not caused by type b organisms.

β-Lactamase (choice A) is an important pathogenic feature of Moraxella catarrhalis, which is


another
important cause of otitis media, but would not be an agent of epiglottitis or meningitis.

IgA protease (choice B) is produced by Streptococcus pneumoniae and Neisseria meningitidis. Both
of these
cause meningitis, but not as commonly in this age group, and would not be the most common causes
of otitis
media in this case.
Lipopolysaccharide (choice C) is present in all gram-negative bacteria and would not be a
distinguishing feature
between those that cause otitis media and epiglottitis.

Nonpilus adhesins (choice D) are mediators of attachment to the epithelium and colonization of the
oropharynx,
but would not be a primary difference between the agents of otitis media and epiglottitis.

Pili (choice E) also mediate attachment to the oropharynx, but would not be the major difference
between the
agents of otitis media and epiglottitis.

Pneumolysin (choice F) is a cytotoxin produced by S. pneumoniae that destroys ciliated epithelial


cells.

Which of the following organisms is most likely to be implicated as a cause of urethritis that persists after
antibiotic therapy for gonorrhea?

A. Actinomyces

B. Chlamydia

C. Mycobacteria

D. Nocardia

E. Rickettsia

Explanation:

The correct answer is B.Chlamydia, Mycoplasma, and Ureaplasma are not effectively treated by
penicillins and
cephalosporins, and are important causes of post-gonococcal urethritis. Chlamydial urethritis can be
diagnosed
by using fluorescent antibodies to identify inclusions in epithelial cells.

Actinomyces(choice A) is a mouth commensal that rarely causes a deeper oral infection.

Mycobacteria(choice C) cause chronic granulomatous diseases such as tuberculosis and leprosy.

Nocardia(choice D) can cause necrotizing pneumonia and disseminated disease.

Rickettsia(choice E) cause typhus and Rocky Mountain spotted fever.

Which of the following is more frequently associated with Klebsiella pneumoniae than with Pseudomonas
aeruginosa?

A. Artificial ventilation
B. Cystic fibrosis

C. Diabetes mellitus

D. Green-colored sputum

E. Upper lobe cavitation

Explanation:

The correct answer is E.Klebsiella pneumoniae is a well-recognized cause of community-acquired


lobar
pneumonia associated with cavitation. It is found typically in alcoholic males over 40 years of age
with
underlying diabetes or obstructive lung disease. Klebsiella pneumoniae mimics Streptococcus
pneumoniae as a
pulmonary pathogen except that Klebsiella has a greater tendency to progress to lung abscess and
empyema.
Pseudomonas aeruginosa is usually associated with patients on ventilators, particularly in intensive
care units.
Immunocompetent patients usually have bilateral bronchopneumonia without cavitary lesions.

Artificial ventilation (choice A) is classically associated with P. aeruginosa infection. The organism
thrives in a
wet environment such as respirators, cleaning solutions, disinfectants, sinks, vegetables, flowers,
endoscopes,
and physiotherapy pools.

P. aeruginosa is a very important pathogen. Mucoid strains of this organism infect the airways in
patients with
cystic fibrosis (choice B), leading to acute exacerbations and chronic progression of lung damage.

Both organisms cause disease in association with diabetes (choice C). Klebsiella pneumoniae
produces
pulmonary disease and P. aeruginosa causes necrotic skin ulcers in diabetics.

Green-colored sputum (choice D) is associated with P. aeruginosa, since more than half of the clinical
isolates
produce the blue-green pigment pyocyanin, which is helpful in identifying the organism.

A 23-year-old woman presents to the emergency room with pelvic pain. A Gram's stain of her cervical
discharge
reveals multiple polymorphonuclear leukocytes, but none contain gram-negative diplococci. Which of
the following
statements best describes the two organisms that most commonly cause this disorder?

A. Both are unlikely to recur because of acquired cell mediated immunity

B. Both are unlikely to recur because of antibody mediated immunity

C. Both induce endocytosis by epithelial cells


D. Both are obligate intracellular parasites

E. Both respond to β-lactam antibiotics

Explanation:

The correct answer is C. In young women, the most likely causes of cervicitis and pelvic
inflammatory disease
(PID) are Neisseria gonorrhoeae and Chlamydia trachomatis. Gram's staining alone may not be able
to
distinguish between the two in women, so culture is warranted. Both organisms induce endocytosis by
epithelial
cells.

Neisseria gonorrhoeae is a gram-negative diplococcus. It is endocytosed by mucus-secreting epithelia,


and is
exocytosed into subepithelial tissues leading to necrotizing acute inflammation and destruction of
ciliated and
non-ciliated cells. Through direct extension, it can lead to PID and infertility in women. It can be
detected by
visualizing the organism in polymorphonuclear leukocytes (PMNs) in Gram's stained clinical material
more easily
in men than in women. Therefore, culture onto Thayer-Martin agar is usually required for diagnosis in
women.
Since the organism can be cultured, it is not an obligate intracellular parasite (compare with choice
D). The
organism can be killed by PMNs, antibody, and complement, however, many of its outer membrane
proteins
undergo antigenic or phase variation or cause the production of blocking antibody that interferes with
bactericidal activity. Immunity to the organism does not develop (compare with choices A and B).
Neisseria
gonorrhoeae is resistant to penicillin, a β-lactam antibiotic (compare to choice E), because of a
plasmid-mediated penicillinase and a chromosomally-mediated mutation that decreases the
permeability of the
outer membrane and decreases the affinity of penicillin binding proteins. It can be treated with
ceftriaxone, a
cephalosporin, which resists the penicillinase, or with quinolones and azithromycin.

Chlamydia trachomatis is an obligate intracellular parasite (choice D) that induces endocytosis and
resides in
the phagosome of infected epithelial cells. It possesses a gram-negative envelope but lacks
peptidoglycan, and
hence is intrinsically resistant to all antibiotics that inhibit peptidoglycan synthesis, such as the
β-lactam
antibiotics (compare with choice E). Antibodies are ineffective at controlling infection. Cell mediated
immunity
(choice A) is probably the major means of controlling this infection since a deficiency in cell
mediated immunity
both increases susceptibility and severity in animal models. Chronic and clinically latent infections
cause serious
morbidity, including pelvic inflammatory disease, fallopian tube dysfunction and blindness.
Chlamydia infection
can be treated with macrolides, quinolones and tetracyclines.
While playing in the park, an 8-year-old girl is bitten in the leg by a neighbor's cat. She presents the next
day with
fever and bone pain localized to her right calf. X-ray reveals a lytic lesion of the right tibia. Results of
the bone
culture are pending. You expect the infecting organism to be

A. Brucella melitensis

B. Eikenella corrodens

C. Francisella tularensis

D. Pasteurella multocida

E. Yersinia pestis

Explanation:

The correct answer is D. This patient has osteomyelitis due to a cat bite that penetrated the
periosteum.
Whenever you see dog or cat bites in a question stem, consider Pasteurella multocida as a primary
cause of
wound infection. This organism is a short, encapsulated, gram-negative rod demonstrating bipolar
staining.
Rapidly arising cellulitis is particularly indicative of this organism.

Brucella melitensis(choice A) is a small, acapsular, gram-negative rod that causes brucellosis


(undulant fever)
and is associated with contact with goats or sheep. Brucella abortus and Brucella suis are variants
associated
with contact with cows and pigs, respectively. The organisms enter the body through the skin or
through
contaminated dairy products, such as unpasteurized imported goat's milk or cheeses.

Eikenella corrodens(choice B) is commonly found in human bites. It is a gram-negative rod that is


part of the
normal flora of the human mouth.

Francisella tularensis(choice C) is a small, pleomorphic, gram-negative rod that causes tularemia


(rabbit fever).
It occurs most commonly in rural areas. In the USA, rabbits are the main reservoir for this organism,
which is
transmitted to humans by the Dermacentor tick or by contact with infectious animal tissues.

Yersinia pestis(choice E) is responsible for bubonic plague, which has been known to occur in the
western USA.
Its main reservoir is the prairie dog, and its vector is the rat flea.

A 6-week-old infant is brought to the emergency room with a 10-day history of coughing and choking
spells. The
white blood cell count is elevated with 80% lymphocytes. The child is gasping for breath,
experiencing paroxysms
of coughing, and vomits twice in the examination room. Encapsulated, gram-negative rods grow out
on
Bordet-Gengou media. Which of the following microorganisms is the likely cause of these symptoms?

A. Bordetella pertussis

B. Haemophilus influenzae type b

C. Klebsiella pneumoniae

D. Legionella pneumophila

E. Mycoplasma pneumoniae

Explanation:

The correct answer is A. The child has whooping cough, or more accurately if one goes by the
chronology of
the symptoms, "coughing whoop," as the patient is subject to a series of coughing episodes that are
followed by
a rapid inspiration of air; the "whoop" is caused by the rapid passage of air through a narrowed
airway. The
infection of the respiratory tract by Bordetella pertussis causes a hyperreactivity of the respiratory
apparatus;
even the slightest stimulus will trigger a coughing episode. The offending pathogen excretes
adenylate cyclase
and also produces an exotoxin that inactivates the inhibitory subunit of the G-protein complex, thus
activating
adenylate cyclase within the cells of the respiratory system. Bordet-Gengou agar is the blood-enriched
(15% vs.
5% for normal blood agar) medium used for the laboratory isolation of this pathogen. Fluorescent
antibody
testing is used to verify the diagnosis.

Haemophilus influenzae type b (choice B) was the major cause of infant meningitis before the Hib
conjugate
vaccine nearly eradicated this pathogen from the United States. Nonencapsulated strains cause otitis
media in
children and pneumonia in adults. This organism also has a peculiar growth requirement, requiring
factor X
(hematin) and factor V (NAD).

Klebsiella pneumoniae(choice C) is a gram-negative, highly encapsulated rod that is a significant


pulmonary
pathogen in individuals with a compromised respiratory apparatus. It is a common cause of aspiration
pneumonia and pulmonary abscesses in alcoholics and patients with chronic obstructive pulmonary
disease.
The organism is readily grown on standard laboratory media such as blood agar or MacConkey's
enteric agar.

Legionella pneumophila(choice D) is another fastidious, gram-negative, respiratory pathogen that


may cause
either a fulminating disease or a mild "walking pneumonia-like" condition (i.e., an atypical
pneumonia). The
organism can be cultured on a charcoal yeast extract medium, but identification is usually
accomplished by
immunofluorescent examination of the pulmonary specimen.

Mycoplasma pneumoniae(choice E) is the most common cause of primary atypical pneumonia. The
disease is
considered "atypical" because the patients have a very mild disease with low-grade fever, little in the
way of
constitutional signs, and a non-productive cough. These organisms are fastidious and are difficult to
grow in the
laboratory. Diagnosis is by immunofluorescent staining or by demonstration of cold hemagglutinins
and strep
MG agglutinins in the serum of the patient.

A patient is admitted to a psychiatric hospital after having been picked up by the police for making
inappropriate
sexual advances. A detailed psychiatric interview demonstrates deficits in memory, insight,
judgement, personal
appearance, and social behavior. The patient is witnessed experiencing a possible epileptic seizure.
Over a
period of several years, motor findings also develop, including relaxed, but expressionless facies,
tremor,
dysarthria, and pupillary abnormalities. Which of the following tests performed on his cerebrospinal
fluid would
most likely be diagnostic?

A. CSF glucose

B. FTA-ABS

C. Gram's stain

D. Lymphocyte count

E. Neutrophil count

Explanation:

The correct answer is B. The patient has neurosyphilis, specifically general paresis, a term that means
"general
paralysis of the insane." In this late sequela of syphilitic infection which occurs 5 to 20 years after
infection,
patients develop mental deterioration, which precedes motor system deterioration, leading eventually
to
"general paralysis" with mutism and incontinence. The abnormalities may be conveniently recalled
using
paresis as a mnemonic for personality, affect, hyperactive reflexes, Argyll Robertson pupils in the
eyes, defects
in the sensorium, intellectual decline and deficient speech. Specific anti-treponemal tests such as
FTA-ABS are
usually positive on both serum and cerebrospinal fluid.

The cerebrospinal (CSF) glucose (choice A) in neurosyphilis is usually normal.

Gram's stain (choice C) of CSF will not demonstrate spirochetes in neurosyphilis.

The CSF lymphocyte count (choice D) is typically elevated in neurosyphilis, but this is a non-specific
finding.

The CSF neutrophil count (choice E) is usually normal in neurosyphilis.

A 65-year-old man develops a vesicular rash localized to a narrow circumferential band on one side of his
chest.
The rash is very painful, and the vesicles are confluent with some ulceration. No other significant
findings are
demonstrated on physical examination. Which of the following diagnoses is most likely to be correct?

A. Chicken pox

B. Herpes simplex I infection

C. Herpes simplex II infection

D. Measles

E. Shingles

Explanation:

The correct answer is E. This is shingles, the recurrent form of herpes zoster infection, which is
usually (except
in the case of immunosuppressed patients) confined to a single dermatome. Isolated vesicles may be
seen
outside the dermatome. The primary herpes zoster infection precedes the development of shingles by
years or
decades; the prevalence of shingles rises steadily with age, to the point that 1% of people older than
80 years
have the condition. Shingles lesions are infections and should be considered an infectious hazard in
the
hospital setting. Acyclovir can ameliorate the condition.

Varicella, or chicken pox (choice A), is the primary form of herpes zoster infection and affects face
and trunk
diffusely.

Herpes simplex I (choice B) affects oral and perioral sites.

Herpes simplex II (choice C) primarily affects genital sites.

Measles (choice D) causes a blotchy rash, rather than a dermatomal one.


The electron transport system of Neisseria is located on which of the following structures?

A. Cytoplasmic membrane

B. Mesosome

C. Mitochondria

D. Nuclear membrane

E. Polyribosome DNA aggregates

Explanation:

The correct answer is A. Unlike eukaryotic cells in which the electron transport system is located on
mitochondria, the electron transport system of bacteria is located on the cytoplasmic (plasma)
membrane.

Mesosomes (choice B) and polyribosome DNA aggregates (choice E) are also found in bacterial cells
and
function in cell division and protein synthesis, respectively.

Bacteria do not have mitochondria (choice C) or membrane-bounded nuclei (choice D).

A Pap smear from a 30-year-old woman demonstrates protozoal parasites. These organisms are likely to be

A. Cryptosporidium parvum

B. Entamoeba histolytica

C. Giardia lamblia

D. Isospora belli

E. Trichomonas vaginalis

Explanation:

The correct answer is E.Trichomonas vaginalis (note "vagina" in the name) is the only organism listed
to
primarily affect the genital tract rather than the intestinal tract. The vaginitis it causes is characterized
by a
frothy, yellow discharge. The organism can be identified in Pap smears, or, more reliably, by special
culture
techniques that are becoming more widely available. Infection in men is usually asymptomatic, but
prostatitis
and urethritis can also occur. Treatment with metronidazole is indicated for both the patient and the
sexual
partner(s).
The other organisms listed (choices A, B, C, and D) cause gastrointestinal disease.

A previously healthy European child who had been diving into a lake develops meningoencephalitis. The
disease
is rapidly fatal, despite aggressive medical therapy. Which of the following organisms is most likely to
be
responsible for this child's illness?

A. Acanthamoeba sp.

B. Balantidium coli

C. Entamoeba histolytica

D. Giardia lamblia

E. Naegleria fowleri

Explanation:

The correct answer is E. Two free-living amoeba have a specific propensity for causing
meningoencephalitis:
Naegleria fowleri and Acanthamoeba sp. The former is seen in the clinical setting described in the
question
stem, and the infection appears to develop when amoebae in lake water are forced across the
cribriform plate
in the nose during swimming and particularly diving. Since the olfactory bulbs are actually part of the
brain,
crossing the cribriform plate allows the amoeba access to the meninges and brain. Diagnosis can be
difficult
because the causative organism may resemble human cells when seen in cerebrospinal fluid.
Unfortunately,
treatment is also difficult because the organism is not very sensitive to presently available anti-
protozoan drugs.

Acanthamoeba sp. (choice A) can also cause meningoencephalitis, but the usual setting is in
immunosuppressed patients.

Balantidium coli(choice B) is a ciliated organism that occasionally causes dysentery resembling that
caused by
Entamoeba histolytica.

Entamoeba histolytica(choice C) causes amoebic dysentery and liver abscesses.

Giardia lamblia(choice D) is a flagellated organism that infects the small intestine, causing diarrhea.

A sexually active 18-year-old woman presents with a fever of 102 F for the past 24 hours and lower
abdominal
pain and anorexia for the past 5 days. On physical examination, there is generalized tenderness of the
abdomen,
and the cervix is erythematous with motion tenderness. There is no rash nor any lesions on the
external genitalia.
A smear of the odorless cervical discharge contains sloughed epithelial cells and scant neutrophils.
Which of the
following would likely be found in the exudate?

A. A naked, icosahedral double-stranded circular DNA virus

B. Iodine-staining intraepithelial inclusion bodies

C. Intraneutrophilic gram-negative diplococci

D. Intranuclear "owl's eye" inclusion bodies

E. Lactose-fermenting gram-negative bacilli

F. Pear-shaped flagellated protozoa

G. Pleomorphic, gram-negative rods

H. Spirochetes on dark-field microscopy

Explanation:

The correct answer is B. The presentation is typical for pelvic inflammatory disease (PID). Chlamydia
trachomata (serotypes D-K) is the most common bacterial cause of sexually transmitted disease
(STD) in this
country and is the most likely agent on the list to produce the symptoms described. It is an ATP-
defective
organism that must therefore live intracellularly in the human host and can be visualized inside
epithelial cells
with iodine, Giemsa, or fluorescent-antibody stains. The remainder of the answer choices refer to
other agents
that could be found in the female genital tract, either by sexual transmission or by contamination with
fecal flora,
but they are not the best choices.

A naked, icosahedral double-stranded circular DNA virus (choice A) refers to human papilloma virus,
which is
the most common cause of STDs in the U.S., but presents with anogenital warts.

Intraneutrophilic gram-negative diplococci (choice C) refers to Neisseria gonorrhoeae, which would


be expected
to present with dysuria and neutrophilic exudate.

Intranuclear "owl's eye" inclusion bodies (choice D) refers to cytomegalovirus, a common STD in the
United
States, but not a common agent of PID. Most cases in average adults are manifested by
mononucleosis-like
symptoms.

Lactose-fermenting gram-negative bacilli (choice E) would be consistent with Escherichia coli.


Although this
organism is the most common cause of urinary tract infections in women in the United States, it
would not be
expected to cause PID.

Pear-shaped flagellated protozoa (choice F) refers to the protozoan parasite Trichomonas vaginalis,
the only
protozoan STD in the world. Infection would be characterized by a malodorous, cheesy exudate, and
there
would be more erythema of the external genitalia than of the cervix.

Pleomorphic, gram-negative rods (choice G) are consistent with Hemophilus ducreyi, which causes
chancroid,
and presents in a different manner.

Spirochetes on dark-field microscopy (choice H) refers to Treponema pallidum, the causative agent of
syphilis,
which would produce rash and/or chancre, depending on the stage of the infection.

Which of the following microorganisms is an obligate aerobe?

A. Bacteroides fragilis

B. Clostridium perfringens

C. Escherichia coli

D. Mycobacterium tuberculosis

E. Pseudomonas aeruginosa

Explanation:

The correct answer is D.Mycobacterium tuberculosis is an important obligate aerobe to remember for
USMLE
Step 1. (Other obligate aerobes include all other Mycobacteria, Bordetella pertussis, and Francisella
tularensis.). This organism can obtain energy only from respiration and must use oxygen as the
terminal
electron acceptor. Primary infection occurs in the upper lobes of the lung.

Bacteroides fragilis(choice A) is an obligate anaerobic gram-negative bacillus that constitutes the


primary
organism found in the colon. It is the most common cause of anaerobic infections, including
abdominal
abscesses and peritonitis.

Clostridium perfringens(choice B), along with all the other Clostridium species, is an obligate
anaerobic,
spore-forming, gram-positive bacillus. C. perfringens is an important cause of infection following
trauma and
surgery and can cause gas gangrene (which can be life-threatening if not treated). C. perfringens can
also
cause food poisoning.
E. coli(choice C) and P. aeruginosa(choice E) are both facultative organisms, meaning that they can
grow both
in the presence or absence of oxygen. E. coli can ferment under anaerobic conditions or respire using
oxygen
as the terminal electron acceptor. P. aeruginosa cannot ferment. It respires only, using nitrate as the
terminal
acceptor under anaerobic conditions and oxygen as the terminal electron acceptor under aerobic
conditions.

A 27-year-old woman presents to the emergency department complaining of 10-12 episodes of nonbloody
diarrhea per day for the past 2 days, along with severe abdominal cramps, nausea, vomiting, and a
low-grade
fever. She states that she just returned from a vacation to Mexico. While in Mexico, she did not drink
any of the
local water and ate only cooked foods and a few fresh salads. If fecal leukocytes are present, the
patient should
most likely be empirically treated with

A. acyclovir

B. ciprofloxacin

C. mebendazole

D. quinine

E. tetracycline

Explanation:

The correct answer is B. When individuals travel from one country to another with marked differences
in climate
and sanitation standards, the risk for development of traveler's diarrhea is dramatically increased. This
type of
diarrhea is most likely to develop within 2-10 days after ingesting local water and/or eating fresh fruits
and
vegetables (such as a fresh salad) that could have been "washed off" with the local water. Traveler's
diarrhea is
often accompanied by the appearance of approximately 10 or more episodes of diarrhea per day, as
well as
severe abdominal cramps, nausea, vomiting, and a mild fever. Depending on the causative bacteria,
blood
and/or fecal leukocytes may be present. Most cases of traveler's diarrhea are caused by enterotoxigenic
Escherichia coli, Shigella species, and Campylobacter jejuni. The most commonly used agents to treat
traveler's
diarrhea are the fluoroquinolones, such as ciprofloxacin, ofloxacin, and norfloxacin.
Trimethoprim/sulfamethoxazole is most commonly used in treatment of children. None of the other
agents are
indicated for the treatment of traveler's diarrhea.

Acyclovir (choice A) is an antiviral agent indicated for the treatment of herpes virus infections.
Mebendazole (choice C) is a broad-spectrum anthelminthic indicated for the treatment of "worm"
infections, such
as Enterobius vermicularis, Trichuris trichiura, and Ascaris lumbricoides.

Quinine (choice D) is an antimalarial agent indicated for the treatment of malaria and severe leg
cramps.

Tetracycline (choice E) is an antibacterial agent with a limited antibacterial spectrum; it is primarily


used in the
treatment of acne vulgaris and gonorrhea infections.

Which of the following types of viruses would be most likely to undergo an abrupt, major antigenic shift
permitting
reinfection of previously exposed individuals?

A. Coxsackie viruses

B. Hepadna viruses

C. Herpesviruses

D. Orthomyxoviruses

E. Paramyxoviruses

Explanation:

The correct answer is D. The phrase "antigenic shift" should tip you off to look for "influenza
viruses" or
"orthomyxoviruses" in the answers. The orthomyxoviruses include influenza viruses A, B, and C.
These viruses
are successful at reinfecting previously exposed individuals because their genome consists of about
eight (the
viral packaging process is a little vague and sometimes entraps more pieces) separate segments of
RNA. Minor
antigenic changes are frequent and are called antigenic drift. Additionally, major changes, called
antigenic shift
rather than antigenic drift, rarely occur, possibly as a result of double infection of cells by human and
animal
influenza viruses with resulting accidental exchange of whole RNA segments. Such dramatic
antigenic shifts
have occurred four times for influenza A since 1933. The other viruses listed in the answers do not
undergo
such dramatic shifts.

A young mother takes her baby to the pediatrician for the first time. The doctor notices the infant's teeth
have
yellow discolorations. The antibiotic this mother most likely took during pregnancy
A. inhibits aminoacyl-tRNA binding

B. inhibits peptidyl transferase

C. interferes with cell wall synthesis

D. is a large, cyclic, lactone-ring structure

Explanation:

The correct answer is A. This question relates to a USMLE favorite side effect—the teeth
mottling that
occurs when a child is exposed to tetracycline in utero. You should remember that tetracycline is
contraindicated in pregnancy and early childhood. Tetracycline is a bacteriostatic drug that binds to
the 30s
subunit of ribosomes, preventing aminoacyl-tRNA from binding with complementary mRNA. This
inhibits peptide
bond synthesis. Resistance is plasmid mediated.

Inhibition of peptidyl transferase (choice B) occurs with chloramphenicol, a broad-spectrum


bacteriostatic agent
that binds to the 50s subunit of ribosomes. Resistance is plasmid mediated. It has high toxicity (GI
disturbances,
aplastic anemia, and gray baby syndrome), so it is used mainly in severe infections or as a topical
agent.

Interference with cell wall synthesis (choice C) occurs with penicillins and cephalosporins, the beta-
lactam
antibiotics. Resistance to these drugs appears in organisms that have developed beta-lactamases
(penicillinases), enzymes that destroy the beta-lactam ring of these medications. The wider spectrum
ampicillin,
amoxicillin, ticarcillin, and carbenicillin are particularly penicillinase susceptible.

Large, cyclic, lactone-ring structures (choice D) describe the macrolides: erythromycin, azithromycin,
and
clarithromycin. They inhibit bacterial protein synthesis by reacting with the 50s ribosomal subunit
and
preventing the release of the uncharged tRNA. Resistance is plasmid mediated. Common side effects
include GI
irritation, skin rashes, and eosinophilia. Erythromycin is a popular choice for patients with penicillin
hypersensitivity. It is a cytochrome p450 inhibitor and therefore must be used with caution in patients
taking
other drugs.

A 7- year-old girl develops behavioral changes, and her performance in school begins to deteriorate.
Several
months later she develops a seizure disorder, ataxia, and focal neurologic symptoms. She is
eventually
quadriparetic, spastic, and unresponsive. Death occurs within a year. This patient may have had which
of the
following viral diseases at 1 year of age?

A. Chickenpox

B. German measles

C. Measles

D. Mumps

E. Parvovirus B19

Explanation:

The correct answer is C. The child has subacute sclerosing panencephalitis (SSPE), which is
fortunately a very
rare, late complication of measles infection at an early age. The existence of this complication is part
of the
rationale to immunize children to measles at an early age. It is thought that very young children's
immune and
neurologic systems may permit the virus to become established in the brain. The exact mechanism of
injury is
poorly understood, but the brain shows encephalitis involving both gray and white matter. 40% of
cases die
within 1 year, and it is thought that the disease is probably always eventually fatal.

None of the other diseases progresses to SSPE.

A 32-year-old man from Yuma, Arizona presents to a hospital with complaints of a 4-day history of fever,
myalgias, and cough. He owns an automobile repair shop and had recently cleaned his garage, which
was
infested by mice. On physical examination, he is tachypneic and must use accessory respiratory
muscles to
breathe. Shortly following admission, he is intubated and diagnosed with adult respiratory distress
syndrome.
What underlying infection should be considered?

A. California encephalitis

B. Cytomegalovirus

C. Hantavirus

D. Poliovirus

E. Rabies

Explanation:
The correct answer is C. Hantavirus pulmonary syndrome, first reported in 1993, consists of fever,
myalgia, and
rapid development of respiratory failure leading to death. The initial cases were mostly among the
Navajo
Indians; however, cases have been documented in many states and in Canada. It is most common in
New
Mexico, Arizona, Colorado, and Utah. The causative agent is a Hantavirus belonging to the family
Bunyaviridae.
The virus is now called Sin Nombre (meaning "without a name") virus. Infection is thought to be
through
inhalation of aerosolized secretions from the common deer mouse. The disease typically begins with a
non-specific prodrome (fever, generalized myalgia, and gastrointestinal disturbances) followed 4 to 5
days later
by respiratory symptoms (cough, dyspnea, and tachypnea). This progresses rapidly to an adult
respiratory
distress syndrome and, in many cases, death.

California encephalitis (choice A) is caused by an arbovirus transmitted by infected mosquitoes. Most


cases are
in children less than 10 years of age. Signs and symptoms include fever, headache, photophobia,
nausea, and
vomiting. Myalgias and arthralgias are typically present. Seizure activity is most frequently observed
in infants
and young children. Most cases are in the midwestern U.S.

Cytomegalovirus (choice B) is a herpesvirus producing a variety of infections. Primary CMV


infection is usually
asymptomatic in immunocompetent patients, but can cause a heterophile-negative mononucleosis
syndrome.
Perinatal infections can occur in utero, intrapartum, or postpartum and can produce congenital
malformations.
CMV can be transmitted by granulocytes in blood transfusions. In patients with AIDS or other
immunosuppressive states, CMV can produce severe disease, including retinitis, pneumonia,
encephalitis,
adrenalitis, and gastrointestinal tract ulcerations.

Poliovirus (choice D) is a member of the Picornavirus family. It remains epidemic in parts of Asia
and Africa even
though the wild-type has been eliminated from the western hemisphere. The virus affects cranial
nerve nuclei
and anterior horn motor neurons of the spinal cord, producing a flaccid paralysis, which is usually
asymmetric.
Vaccine-related polio can occur with the live virus vaccine.

Rabies (choice E) is a Rhabdovirus that produces encephalitis and/or myelitis. It should be considered
in
persons who have recently traveled outside of the U.S. The most common sources of exposure are
dogs, cats,
skunks, foxes, raccoons (in Florida and Connecticut), wolves, and bats. The infection is acquired by
the bite of
a rabid animal or by inhalation. Rabies also has been reported to occur in patients after corneal
transplantation.
The virus spreads along peripheral nerves to the central nervous system. Symptoms include
hydrophobia and
copious salivation.
A 55-year-old man returns from a vacation to the beach. While on vacation, he and his family consumed
several
meals consisting primarily of seafood, including lobster and raw oysters. Although the other members
of his family
did not get sick, he developed a bullous rash over his lower extremities, hypotension, and confusion.
His past
medical history is significant for liver disease and alcoholism. The most likely cause of this man's
signs and
symptoms is

A. Aspergillus fumigatus

B. Campylobacter jejuni

C. Candida albicans

D. Streptococcus (Group A)

E. Vibrio vulnificus

Explanation:

The correct answer is E.Vibrio vulnificus septicemia is contracted by consuming raw oysters. Patients
with
chronic renal insufficiency, liver disease, hematopoietic disorders, and a past history of alcoholism
are
particularly at risk. His past medical history makes him more susceptible to septicemia than are his
family
members. Septicemia cause by Vibrio vulnificus generally begins with chills, fever, and hypotension,
and skin
lesions tend to occur 24-48 hours after the onset of the infection.

Aspergillus fumigatus(choice A) is an opportunistic mold responsible for infections of wounds and


burns in
immunocompetent hosts; in immunocompromised hosts, it can invade visceral organs such as the
lungs.

Campylobacter jejuni(choice B) are curved, gram-negative rods that cause enterocolitis with diarrhea,
and, less
commonly, chronic gastritis.

Candida albicans(choice C) is a common opportunistic yeast that is found as normal flora of upper
respiratory,
gastrointestinal, and vaginal mucosae. It ordinarily causes thrush and vulvovaginitis; it may
disseminate in the
immunocompromised patient.

Group A streptococcus (Streptococcus pyogenes; choice D) is an important cause of bacterial


pharyngitis
(Strep throat).

A 24-year-old female has fever, malaise, and a dry, nonproductive cough. She also complains of headache,
muscle aches, and leg pain. Lab values are significant for elevated cold agglutinins. Which of the
following
microorganisms is responsible for her symptoms?

A. Haemophilus influenzae

B. Klebsiella pneumoniae

C. Legionella pneumophila

D. Mycoplasma pneumoniae

E. Streptococcus pneumoniae

Explanation:

The correct answer is D.Mycoplasma pneumoniae is a wall-less bacterium that causes interstitial
pneumonia in
young adults. Elevated cold agglutinins (a classic clue) are found in about half of the patients. The
cold
agglutinins are IgM antibodies. M. pneumoniae may be diagnosed by sputum or complement fixation.

Haemophilus(choice A) causes bronchopneumonia in babies and children, and may occur in


debilitated adults.

Klebsiella(choice B) causes a bronchopneumonia with patchy infiltrates involving one or more lobes.
Think red
currant jelly sputum. It frequently occurs in debilitated patients, diabetics, and alcoholics. Note that
these
organisms are highly encapsulated and produce mucoid colonies on lab media.

Legionella(choice C) lives in contaminated water sources such as air conditioning systems.

Streptococcus pneumoniae(choice E) is the most common cause of pneumonia in the elderly


population and in
those with poor nutrition. It may lead to a pleural abscess and rusty-colored sputum; it is also a
common cause
of sepsis and meningitis in the elderly.

A Native American man is brought to a rural hospital in New Mexico. On arrival, he is unconscious with
severe
bronchopneumonia. Family members state that he suffered the sudden onset of chills, fever, and
headache
several days ago. One day later, the man complained of chest pain and difficulty breathing, and
coughed up
blood-tinged sputum. Chest x-ray reveals patchy infiltrates and segmental consolidation. Which of the
following
organisms is the most likely cause of this man's pneumonia?
A. Brucella abortus

B. Clostridium perfringens

C. Francisella tularensis

D. Listeria monocytogenes

E. Yersinia pestis

Explanation:

The correct answer is E. Any previously healthy person in the Southwestern United States who
develops septic
shock or severe pulmonary disease should be evaluated for plague. Plague is not an extinct disease,
but is still
encountered in sporadic cases in various places in the world, including Asia, Africa, parts of Europe,
and the
American Southwest. The causative organism is Yersinia pestis, which is endemic in many wild
animal
populations, and can be transmitted to humans either by direct contact or by arthropod bite. Human
plague may
take many forms, including pestis minor (mild lymphadenopathy); bubonic plague (prominent
lymphadenopathy); pneumonic plague (as in this patient); and septicemic plague. The primary
pneumonic form
typically presents as described. Antibiotics are most effective if given within the first 24 hours, which
can be
problematic if medical staff do not suspect the disease. Since plague is rare in the United States, a
high degree
of clinical suspicion is required to make a rapid diagnosis and to institute timely treatment. If the
diagnosis is
missed, the mortality rate is quite high.

Brucella abortus(choice A) causes brucellosis, characterized by undulating fever, lymphadenopathy,


and
hepatosplenomegaly.

Clostridium perfringens(choice B) causes gas gangrene and gastroenteritis.

Francisella tularensis(choice C) causes tularemia, associated with a spectrum of manifestations from


an
influenza-like syndrome to adenopathy with ulceration at the site of inoculation.

Listeria monocytogenes(choice D) causes listeriosis. Infection during pregnancy may result in sepsis,
abortion
or premature delivery. Infection in the neonate may produce meningitis. In immunocompromised
adults, either
meningitis or sepsis may occur.

An elderly patient has pneumococcal pneumonia. She has no known drug allergies. Which of the following
antibiotics would be most appropriate?

A. Chloramphenicol
B. Cefotaxime

C. Erythromycin

D. Penicillin

E. Vancomycin

Explanation:

The correct answer is D. Penicillin remains the first-line drug of choice for pneumococcal pneumonia,
except in
those patients with penicillin allergy and in the relatively few areas in which pneumococcal strains
with high-level
penicillin resistance are found. Alternative therapies include erythromycin and vancomycin.

Chloramphenicol (choice A) is not usually used for pneumococcal pneumonia.

The third generation cephalosporin cefotaxime (choice B) is not usually used for pneumococcal
pneumonia.

Erythromycin (choice C) is a good alternative therapy for pneumococcal pneumonia, but is usually
used only
when a penicillin allergy is present.

Vancomycin (choice E) is not the first-line therapy, but it is a good alternative in patients allergic to
penicillin or
when high-level penicillin resistance (relatively uncommon) is present.

A 30-year-old veterinarian visits her obstetrician for a first-trimester prenatal check-up. She has no
complaints.
Routine physical exam is significant only for mild cervical lymphadenopathy. She is prescribed
spiramycin but is
noncompliant. Her baby is born with hydrocephalus and cerebral calcifications. Which of the
following organisms is
most likely responsible?

A. Isospora belli

B. Leishmania donovani

C. Plasmodium vivax

D. Toxoplasma gondii

E. Trypanosoma cruzi

Explanation:
The correct answer is D. Humans become infected with Toxoplasma gondii by ingesting cysts in
contaminated
food or through contact with cat feces. The veterinarian in question was therefore particularly at risk
of infection.
T. gondii is especially hazardous in pregnant women because the organism can be transmitted to the
fetus
through the placenta. (It is part of the ToRCHeS group of congenital infections–Toxoplasma,
Rubella,
CMV, Herpes/HIV, Syphilis). Since infected mothers are usually asymptomatic, cases often go
unnoticed.
Occasionally, patients present with cervical lymphadenopathy, as did the veterinarian, and require
treatment to
prevent complications in the fetus. Though newborns are also often asymptomatic, they are at risk for
developing the classic triad of chorioretinitis (at birth or later in life), hydrocephalus, and cerebral
calcifications.
Note that T. gondii is also a common cause of CNS infections (e.g., encephalitis) in HIV-positive
patients.

Isospora belli(choice A) is an intestinal protozoan that causes watery diarrhea, particularly in the
immunocompromised. Fecal-oral transmission of oocysts allows invasion of small intestinal mucosa,
destroying
the brush border.

Leishmania donovani(choice B) causes kala-azar (visceral leishmaniasis), which is characterized by


fever,
weakness, weight loss, splenomegaly, and skin hyperpigmentation. It is prevalent in regions of the
Mediterranean, Middle East, Russia, and China. The vector is the sandfly.

Plasmodium vivax(choice C) causes malaria and is transmitted by the female Anopheles mosquito,
which
introduces sporozoites into the blood. These differentiate into merozoites that destroy erythrocytes.
Splenomegaly ensues. Other species of this organism also cause malaria: P. malariae and P.
falciparum (which
causes a more severe form of the disease). Note that sickle cell trait confers resistance to this disease.

Trypanosoma cruzi(choice E) causes Chagas' disease, characterized by facial edema and nodules,
fever,
lymphadenopathy, and hepatosplenomegaly. It affects cardiac muscle most severely and is a major
cause of
cardiac disease worldwide. The reduviid ("kissing") bug is the vector that infects humans through
bites. It is most
prevalent in Central and South America, with rare cases in the southern US.

A 30-year-old veterinarian on a cattle ranch presents with a 1-to-2-month history of malaise, chills,
drenching
malodorous sweats, fatigue, and weakness. He has anorexia and has lost 15 pounds. He has
intermittent fevers
that range up to 103 F (39.4 C). He complains of visual blurring. A physical examination reveals mild
lymphadenopathy, petechiae, and a cardiac murmur consistent with aortic insufficiency. What is the
most likely
etiologic agent?
A. Bacillus anthracis

B. Brucella abortus

C. Coccidioides immitis

D. Erysipelothrix rhusiopathiae

E. Trichinella spiralis

Explanation:

The correct answer is B.Brucella abortus produces a chronic, granulomatous disease with caseating
granulomas. Most cases occur in four states (Texas, California, Virginia, and Florida), and are
associated with
cattle, in which it produces spontaneous septic abortions. Most cases of brucellosis produce mild
disease or
fevers of unknown origin. However, Brucella spp. can infect the cardiovascular system and cause a
localized
infection. B. abortus is the most common species to cause endocarditis. The aortic valve is most
commonly
involved, followed by the mitral valve, and then both valves. Most cases of brucellosis are associated
with
occupational exposure, in persons such as veterinarians, ranchers, and those who handle carcasses.

Bacillus anthracis(choice A) is the causative agent for anthrax. It usually produces cutaneous disease
(malignant pustule or eschar) at the site of inoculation in handlers of animal skins. It can also produce
a severe
hemorrhagic pneumonia (Woolsorter's disease) and septicemia. At-risk groups include those who
handle animal
carcasses or skins.

Coccidioides immitis(choice C) is a dimorphic fungal disease producing a granulomatous pulmonary


syndrome
that is more severe in dark-skinned individuals. Disseminated disease occurs most often in Filipinos,
Mexicans,
and Africans. The infective form is the arthrospore; the diagnostic form in tissue is the spherule
containing
endospores. The disease is endemic in the San Joaquin River Valley. At-risk groups include military
personnel,
agricultural workers, construction workers, oil field workers, archaeology students, participants in
outdoor
sports, and sightseers. Remote infections from fomites (cotton harvested in the Southwestern U.S.)
have been
reported.

Erysipelothrix rhusiopathiae(choice D) is a pleomorphic, gram-negative rod that causes a localized


skin
infection. It is an occupational disease of fishermen, fish handlers, butchers, meat-processing workers,
poultry
workers, farmers, veterinarians, abattoir workers, and housewives.

Trichinella spiralis(choice E) is a nematode infection caused by the ingestion of larvae found in


undercooked
meat. Pork is the most common contaminated meat. However, outbreaks in the northern parts of the
U.S. have
been associated with eating undercooked infected bear meat. Symptoms include diarrhea, periorbital
edema,
myositis, fever, and eosinophilia.

Six days after receiving several flea bites in a rat-infested shed in Southeastern New Mexico, a homeless
24-year-old-man develops fever, chills, and a rash that spreads from his abdomen to cover his
extremities. He is
seen at the hospital emergency department, where blood is drawn for analysis. Eight days later, the
public health
department reports the presence of antibody to one of the rickettsial group antigens. Which of the
following is
the most likely diagnosis?

A. Endemic typhus

B. Epidemic typhus

C. Q fever

D. Rocky Mountain spotted fever

E. Scrub typhus

Explanation:

The correct answer is A. Endemic typhus is caused by Rickettsia typhi and is found worldwide. It is
spread by
the feces of the rat flea, and its reservoir is the rat.

Epidemic typhus (choice B) is cause by Rickettsia prowazekii. It is prominent during times of war
and social
upheaval and is spread by the feces of the body louse, Pediculus humanis. Humans are the chief
reservoir,
although it is also found in populations of flying squirrels along the Atlantic coast of the U.S.

Q fever (choice C) is caused by Coxiella burnetii of the family Rickettsiaceae. Unlike the other
rickettsial
illnesses, Coxiella burnetii is not usually transmitted to humans by the bite of an arthropod. Since the
organism
is resistant to dehydration, it can be acquired by inhaling dust contaminated by animals with
asymptomatic
infections, such as goats, sheep, and cattle.

Rocky Mountain spotted fever (RMSF) (choice D) is caused by Rickettsia rickettsii. It is transmitted
by the bite
of an infected tick of the genus Dermacentor. Since the tick passes the infection transovarially to its
progeny,
the tick is a reservoir. Rodents and other mammals are also reservoirs. Despite the name, most cases
of RMSF
occur in the central states and southern Atlantic seaboard states. Oklahoma has the highest incidence
of the
disease.

Scrub typhus (choice E) is caused by Rickettsia tsutsugamushi. It occurs mainly in Southeast Asia,
where it is
transmitted by mites. Mites and rodents are the reservoirs for the organism.

A 54-year-old diabetic patient reports to his physician's office complaining of an unresolved skin lesion on
his
foot. The lesion began several weeks ago as a blister and has since become a painful, erosive,
expanding sore.
On examination, the affected site is now 5 cm in diameter, with a black necrotic center and raised red
edges.
Which of the following toxins has a mechanism of action most similar to the toxin responsible for
tissue damage in
this patient?

A. Anthrax toxin

B. Botulinum toxin

C. Cholera toxin

D. Clostridium perfringens alpha toxin

E. Diphtheria toxin

F. Escherichia coli labile toxin

G. Pertussis toxin

H. Shigatoxin

I. Streptococcal erythrogenic toxins

J. Tetanus toxin

K. Toxic shock syndrome toxin-1

Explanation:

The correct answer is E. This patient's wound is infected with Pseudomonas aeruginosa, and the
characteristic
lesion described is called ecthyma gangrenosum. The Pseudomonas alpha toxin, which is responsible
for the
tissue damage, inhibits protein synthesis by acting on EF-2 with a primary target cell in the liver. The
diphtheria
toxin has a similar action, although its target cells are heart and nerve.

Anthrax toxin (choice A) is an adenylate cyclase that causes fluid loss from cells.
Botulinum toxin (choice B) is a neurotoxin that decreases acetylcholine synthesis.

Cholera toxin (choice C) acts to increase adenylate cyclase activity by ribosylation of GTP-binding
protein.

Clostridium perfringens alpha toxin (choice D) is a lecithinase.

Escherichia coli labile toxin (choice F) works in a fashion similar to the cholera toxin.

Pertussis toxin (choice G) causes fluid loss by ribosylating Gi.

Shigatoxin (choice H) decreases protein synthesis by inhibiting the 60S ribosomal subunit.

Streptococcal erythrogenic toxins (choice I) act similarly to the diphtheria toxin, but do so by
increasing cytokine
production.

Tetanus toxin (choice J) is a neurotoxin that inhibits the inhibitory neurotransmitters glycine and
GABA.

TSST-1 (choice K) is a superantigen that acts by increasing cytokine production and decreasing liver
clearance
of endotoxin.

An 18-year-old, previously healthy female presents to the student health service with fever, vomiting, and
diarrhea. On physical examination, she is hypotensive and has an erythematous, red, sunburn-like skin
rash. She
is currently menstruating, and has been using super absorbent tampons. Which of the following
findings from a
positive blood culture would confirm your suspected diagnosis?

A. Organisms are acid-fast

B. Organisms are coagulase positive

C. Organisms grow on EMB (eosin-methylene blue) agar

D. Organisms grow on Thayer-Martin media

E. Organisms have positive Quellung reaction

Explanation:

The correct answer is B. This is a multi-step microbiology question that requires you to diagnose the
illness,
identify the microorganism, and remember its key feature. The first part should be easy: everything
about this
vignette suggests toxic shock syndrome. The organism in question is therefore Staphylococcus
aureus, which is
coagulase positive. All of the other choices are classic features of other important pathogenic
microorganisms:
Acid-fast organisms (choice A), refers to Mycobacteria. (In addition, Nocardia species are partially
acid fast).

EMB agar (choice C) refers to a selective and differential medium used to isolate and identify enteric
gram-negative bacteria. Gram-positive bacteria will not grow on EMB agar because the addition of
eosin inhibits
their growth. Nonlactose fermenters will have colorless colonies, while fermentation of this sugar will
cause the
colonies to appear pink or purple.

Thayer-Martin media (choice D) is a growth medium for pathogenic Neisseria species. It contains the
antibiotic
vancomycin, which kills gram-positive organisms such as Staphylococcus aureus.

The Quellung reaction (choice E) can be used to identify the capsule type of a microorganism.
Encapsulated
microorganisms, like the Pneumococci and Haemophilus, are mixed with specific antisera. If the
antiserum is
directed against the microorganism's capsule type, the capsule will be opsonized, absorb water, and
become
visible under a light microscope.

A 24-year-old woman presents with a 3-day history of fever, chills, chest pain, and cough productive of
rust-colored sputum. Past medical history includes a splenectomy 1 year ago. A chest x-ray film
indicates
consolidation of the right lower lobe. Blood cultures are positive for α-hemolytic gram-positive
diplococci.
Immunity to the causative organism is based on

A. alternative complement pathway activation

B. antibody to an α-helical coiled fimbria

C. IgA antibodies to C carbohydrate

D. IgG antibodies to C carbohydrate

E. IgG antibodies to a surface acidic polysaccharide

Explanation:

The correct answer is E. The patient in this question has pneumococcal pneumonia, which must be
considered in any patient with chills, fever, chest pain, and cough productive of purulent, rust-colored
sputum.
Streptococcus pneumoniae is an α-hemolytic, gram-positive coccus that grows in chains. It can
be easily
distinguished from other α-hemolytic streptococci because it is exquisitely sensitive to bile and
bilelike
compounds, such as optochin. It is the most common cause of community-acquired pneumonia and
the most
common cause of community-acquired meningitis in adults older than 30. The only recognized
virulence factor
of S. pneumoniae is its carbohydrate capsule (which contains acidic polysaccharides). Antibody to a
specific
capsule type is necessary to overcome infection. More than 80 capsule types have been recognized.
The 23
types that most commonly cause disease are contained in a vaccine that is recommended for high-risk
groups,
including the elderly and those undergoing splenectomy. Increased susceptibility is also found in
patients with
Hodgkin disease, chronic lymphocytic leukemia, and myeloma. The vaccine should still be given to
patients with
these conditions, but it is less successful.

The alternative complement pathway (choice A) is important in clearing Neisseria infections.


Individuals with
deficiencies in C5 through C8 are at increased risk of disease from Neisseria.

The fimbria of Streptococcus pyogenes (Group A α-hemolytic streptococcus) is composed of


an
α-helically coiled M protein. Antibody against a specific M type (choice B) will prevent
infection. However,
raising antibodies to M proteins can lead to rheumatic fever, so Strep throat infections are routinely
treated with
penicillin to prevent an antibody response.

The C carbohydrate is an antigen of α-hemolytic streptococci used to divide them into different
groups.
Antibody against C carbohydrate (choices C and D) is not protective.

A 3-year-old boy presents with a 1-day history of loose stools, fever, abdominal cramping, headache, and
myalgia. He has no blood in the stool. A careful history reveals that he has several pet turtles. Which
of the
following is most likely the causative agent of his diarrhea?

A. Chlamydia psittaci

B. Entamoeba histolytica

C. Salmonella spp.

D. Staphylococcus aureus

E. Yersinia enterocolitica

Explanation:

The correct answer is C. Salmonella spp., including S. enteritidis and S. typhimurium, produce a
gastroenteritis
or enterocolitis. Patients with decreased gastric acidity, sickle cell disease, defects in immunity, or
children
younger than 4 years have a more severe course of disease. Salmonella spp. are carried in nature by
animal
reservoirs such as poultry, turtles, cattle, pigs, and sheep. The incubation period is 8-48 hours after
ingestion
of contaminated food or water.

Chlamydia psittaci(choice A) produces an interstitial pneumonitis accompanied by headache,


backache, and a
dry, hacking cough. A pale, macular rash is also found on the trunk (Horder's spots). Patients at risk
include pet
shop workers, pigeon handlers, and poultry workers.

Entamoeba histolytica(choice B) produces a diarrhea (frequently bloody or heme-positive), right


lower quadrant
crampy abdominal pain, and fever. Patients frequently have weight loss and anorexia. There is usually
a history
of travel outside the U.S. Most cases are chronic. Complications include liver abscesses.

Staphylococcus aureus(choice D) produces a self-limited gastroenteritis due to the production of


preformed,
heat-stable enterotoxins. The incubation period is 16 hours. The toxins enhance intestinal peristalsis
and
induce vomiting by a direct effect on the CNS.

Yersinia enterocolitica(choice E) usually produces a chronic enteritis in children. These patients have
diarrhea,
failure to thrive, hypoalbuminemia, and hypokalemia. Other findings include acute right lower
quadrant
abdominal pain, tenderness, nausea, and vomiting. The infection mimics appendicitis or Crohn's
disease.

Bilateral tonsillectomy is performed on an otherwise healthy 11-year-old female with recurrent upper
respiratory
tract infections. On sectioning the tonsils, numerous small, yellow granules are noted. A granule
crushed
between two slides has a dense, gram-positive center and numerous branching filaments at the
periphery. The
granules are most likely composed of which of the following organisms?

A. Actinomyces israelii

B. Aspergillus fumigatus

C. Blastomyces dermatitidis

D. Candida albicans

E. Corynebacteria diphtheriae

Explanation:

The correct answer is A. Actinomyces are normal inhabitants of the gastrointestinal tract that grow
under
anaerobic and microaerophilic conditions. Although they are gram-positive rods, they grow as
branching
filaments and have been confused with fungi. The yellow colonies (sulfur granules) are found in low-
oxygen
niches like the tonsils and in actinomycotic abscesses.

Aspergillus fumigatus(choice B)may be present in the respiratory tract as an opportunistic pathogen;


however,
fungus balls are generally seen only in pre-existing cavities (e.g., bronchiectasis, TB), not in the
tonsils.

Blastomyces dermatitidis(choice C) is a respiratory pathogen that is seen as thick-walled yeasts


within
granulomas.

Candida albicans(choice D), also a normal inhabitant of the oral cavity, would present as whitish
plaques and
would appear microscopically as budding yeasts.

Diphtheria, caused by Corynebacteria diphtheriae(choice E), is a gram-positive rod. The disease


presents with
a gray-white membrane in the oropharynx, and large colonies would not be appreciated.

A patient with a cavitary lung lesion coughs up sputum that contains thin, acid-fast positive rods. Which of
the
following features would most likely be associated with these bacteria?

A. Nutritional requirement for factors V and X

B. Streptokinase

C. Toxic shock syndrome toxin

D. Visible under dark field illumination

E. Waxy envelope

Explanation:

The correct answer is E. Mycobacteria, such as the causative organism of this patient's tuberculosis,
are "acid
fast" because they have an envelope that contains large amounts of lipids and even true waxes (unlike
envelopes of other types of bacteria) that prevents the acid-fast stain (carbolfuchsin) from leaking
out.

The other characteristics listed in the answers are commonly tested features of specific bacteria:

Nutritional requirement for factors V and X (choice A) is a feature of Haemophilus influenzae.

Streptokinase (choice B) is a feature of Streptococci.

Toxic shock syndrome toxin (choice C) is a feature of Staphylococcus aureus.


Visibility under dark field illumination (choice D) is a feature of the syphilis organism Treponema
pallidum.

An 8-year-old boy is brought to the emergency room with a 3-day history of fever to 102 degrees F and
abdominal pain. He also complains of pain in his right knee and right elbow. He was seen four weeks
ago
because of a sore throat and a rash. A throat culture performed at that time grew gram-positive cocci
in chains.
Amoxicillin was prescribed, but the boy's mother did not fill the prescription. On physical exam his
temp is 101.7F,
HR 96, and BP 100/60. Cardiac exam reveals a pansystolic blowing murmur heard best at the apex.
His right
elbow is tender on extension and flexion with mild swelling. Laboratory tests reveal a positive C-
reactive protein,
an ESR of 40 and a WBC of 22,000 with a left shift. EKG shows a prolonged PR interval. Which of
the following
tests would be positive for the microorganism responsible for this patient's illness?

A. Catalase test

B. Coagulase test

C. Sensitivity to bacitracin

D. Sensitivity to novobiocin

E. Sensitivity to optochin

Explanation:

The correct answer is C. This case is classic for Rheumatic fever, including two of the major Jones
criteria
(carditis, polyarthritis) and several minor criteria (fever, arthralgia, elevated ESR, leukocytosis, C-
reactive
protein, prolonged PR interval). Rheumatic fever is a sequela of untreated infection with Group A
Streptococcus (S. pyogenes). S. pyogenes is differentiated from the other beta-hemolytic Strep by its
sensitivity
to the antibiotic bacitracin.

The catalase test (choice A) is used to differentiate Staphylococci from Streptococci. Staphylococci
are
catalase positive, Streptococci are catalase negative. So, S. pyogenes would be catalase negative.

The coagulase test (choice B) is used to differentiate Staph aureus from the other Staph spp. Staph
aureus is
coagulase positive; the others are negative.

Sensitivity to novobiocin (choice D) is used to differentiate Staph saprophyticus (resistant) from


Staph
epidermidis (sensitive).
Sensitivity to optochin (choice E) is used to differentiate Strep pneumoniae (sensitive) from viridans
Strep
(resistant).

An immigrant from the Far East develops malaise, fever, and rigors, followed by upper right quadrant
abdominal
pain, vomiting, jaundice, and itching. His urine is dark and his feces are pale. Infestation with which
of the
following parasites is most strongly suggested by this patient's presentation?

A. Clonorchis sinensis

B. Enterobius vermicularis

C. Plasmodium ovale

D. Taenia solium

E. Trypanosoma cruzi

Explanation:

The correct answer is A. The patient is suffering from suppurative (bacterial) cholangitis, which can
occur as a
complication of infestation by the roundworm, Ascaris lumbricoides and by the liver flukes,
Clonorchis sinensis
and Fasciola hepatica. Biliary tract obstruction produces jaundice and extreme itching, with dark
urine and pale
feces. Therapy typically includes emergency endoscopic sphincterectomy to improve biliary drainage,
antibiotics, and anthelminthic agents.

Enterobius vermicularis(choice B) causes pinworm infections.

Plasmodium ovale (choice C) causes malaria.

Taenia solium (choice D) is the pork tapeworm. Adult tapeworms cause taeniasis, while the larvae are
responsible for cysticercosis.

Trypanosoma cruzi(choice E) causes Chagas' disease.

A 38-year-old woman vacationing in Connecticut is bitten by a tick. She does not seek medical treatment
and
eventually develops chronic arthritis of the knee and hip joints and paralysis of the left facial muscles.
A physical
examination during the early stages of the disorder would most likely have revealed

A. aphthous ulcers in the mouth


B. erythema chronicum migrans

C. flaccid paralysis of limb flexors

D. purpuric lesions in a bathing trunk distribution

E. spastic paralysis of limb extensors

Explanation:

The correct answer is B. Lyme disease should be suspected in a patient who is bitten by a tick in the
northeastern U.S. Lyme disease was named after a township in eastern Connecticut where the disease
was
endemic. The disease is spread via a tick vector of the genus Ixodes, which transmits a spirochete that
causes
a systemic illness. Erythema chronicum migrans is usually the first sign of the illness. This is a large
red patch
on the buttocks or chest that slowly expands as the center blanches. Generally, patients also have
constitutional symptoms, such as fever and chills, during this phase. Stiff neck may develop, along
with other
signs of meningeal irritation, because of an aseptic meningitis. Other neurologic complications of
Lyme disease
include Bell's palsy due to involvement of branches of the facial nerve. Arthritis is a prominent
feature in about
half the patients with Lyme disease. It tends to appear several months after the infection but may
persist for
several years. The course of the chronic arthritis shows exacerbations and remissions; the most
commonly
affected joints are the knees and hips. Cardiac abnormalities in Lyme disease include pericarditis and
heart
block.

Skin manifestations do not include aphthous ulcers (choice A).

Flaccid or spastic paralysis of limbs (choices C and E) does not accompany Lyme disease; neurologic
involvement is generally limited to cranial nerves and meningitis.

Purpura (choice D) is associated with vasculitis and does not occur in Lyme disease.

Global eradication of Lyme disease is unlikely because

A. Borrelia burgdorferi can be maintained in nature indefinitely by a tick vector

B. Borrelia burgdorferi is resistant to antibiotics and disinfectants

C. Borrelia burgdorferi is resistant to environmental stresses

D. human disease may reactivate after the primary infection (Brill-Zinsser disease)

E. humans are the primary reservoir for Borrelia burgdorferi

Explanation:
The correct answer is A.Borrelia burgdorferi can be maintained in nature indefinitely by a tick vector.
This
organism is the tick-transmitted spirochete that causes Lyme disease. The tick, Ixodes dammini, can
infect both
the white-footed mouse and large mammals such as deer during its life cycle, making them reservoirs.
The tick
itself is a reservoir, however, since it acquires the disease through transovarial passage of the
organism.
Together, these factors make Lyme disease an endemic infection with little hope for eradication.

Borrelia burgdorferi is not resistant to antibiotics and disinfectants (compare with choice B). The
spirochete can
be successfully treated with penicillins, tetracycline, and ceftriaxone.

Borrelia burgdorferi is a delicate spirochete and is not resistant to environmental stresses (compare
with choice
C).

Brill-Zinsser disease (choice D) is the reactivation of epidemic typhus infection caused by Rickettsia
prowazekii.
It can occur many years after an infection that was not treated with antibiotics.

Humans are incidental hosts, rather than the primary reservoir, for Borrelia burgdorferi(choice E).
The primary
reservoirs are ticks, mice, and large mammals.

A 54-year-old farmer in rural Pennsylvania presents to his physician with chronic cough. Chest x-ray
demonstrates a mass lesion with hilar lymphadenopathy. Biopsy of the mass demonstrates multiple,
tiny yeast
forms within macrophages. Which of the following is the most likely diagnosis?

A. Blastomycosis

B. Coccidioidomycosis

C. Histoplasmosis

D. Paracoccidioidomycosis

E. Sporotrichosis

Explanation:

The correct answer is C. Histoplasmosis is caused by Histoplasma capsulatum, a dimorphic fungus


that grows as
a mold in the wild, but as a tiny yeast inside macrophages in humans. The disease is typically
asymptomatic or
mild enough to go undetected, but when symptomatic, it presents with cough, fever, and malaise.
Blastomycosis (choice A) is characterized by a larger, round-budding yeast form, seen free in the
tissues.

Coccidioidomycosis (choice B) is seen mainly in the desert parts of the Southwest U. S. In the lungs,
spherules
containing endospores are seen.

Paracoccidioidomycosis is endemic in Latin America, especially in Brazil. The infected cells show a
typical "pilot's
wheel" appearance due to multiple yeasts sprouting out of a single parent cell (choice D).

Sporotrichosis (choice E) most often produces a localized cutaneous infection, following inoculation
occurring in
association with minor skin trauma during gardening.

Two weeks after birth, a neonate develops sepsis, skin vesicles, and conjunctivitis. Over the next several
days,
the baby's condition deteriorates with development of seizures, cranial nerve palsies, and lethargy. The
baby dies
approximately one week after onset of symptoms. Which of the following infectious agents would
most likely cause
this clinical presentation?

A. Cytomegalovirus

B. Herpes simplex

C. Rubella

D. Syphilis

E. Toxoplasmosis

Explanation:

The correct answer is B. All of the agents listed, including choices A, C, D and E, can cause
devastating
congenital infections with high mortality and often with major organ malformation (the TORCH
agents:
Toxoplasma, other, rubella, cytomegalovirus, herpes simplex). However, it is herpes simplex type II,
typically
acquired during delivery, that causes the devastating neonatal encephalitis described in the question
stem. The
mortality rate for neonatal herpes is about 65%, and only 10% of the babies escape without
neurologic
sequelae. A point worth remembering is that adult herpes encephalitis (in non-immunosuppressed
individuals) is
usually due to herpes simplex I, while neonatal and congenital herpes are usually due to herpes
simplex II.
A one-week-old baby develops nuchal rigidity and fever. A lumbar puncture is performed and the
cerebrospinal
fluid demonstrates large numbers of neutrophils. Which of the following is the most likely causative
agent?

A. Coxsackievirus

B. Escherichia coli

C. Herpes virus

D. Mycobacterium tuberculosis

E. Neisseria meningitidis

Explanation:

The correct answer is B. The nuchal rigidity (stiff neck) suggest meningitis. Numerous neutrophils in
the
cerebrospinal fluid suggests a bacterial pathogen. The best answer of those listed is Escherichia coli,
which is
normally a gut organism, but can infect neonates who acquire the organism during passage through
the birth
canal.

Coxsackievirus (choice A) is a cause of acute lymphocytic meningitis.

Herpes virus (choice C) is a cause of lymphocytic meningitis.

Mycobacterium tuberculosis(choice D) is a cause of chronic meningitis.

Neisseria meningitidis(choice E) causes bacterial meningitis, usually in the second or third decade of
life.

A patient presents to a physician because of pain during defecation accompanied by blood in the stool.
Physical
examination demonstrates a large perianal mass. Pathologic examination of the rectal mass following
resection
demonstrates a condyloma in which transformation to frank carcinoma has occurred. Which of the
following
viruses would most likely be associated with these lesions?

A. Epstein-Barr virus (EBV)

B. Hepatitis B virus (HBV)

C. Human herpesvirus type 8 (HHV8)

D. Human papilloma virus (HPV)

E. Human T-cell leukemia virus (HTLV-1)


Explanation:

The correct answer is D. The tumor is anal carcinoma arising in a condyloma. Both condyloma and
anal
carcinoma are related to human papilloma virus (HPV), which is also associated with cervical and
penile
condylomas and carcinomas.

EBV (choice A) is associated with Burkitt's lymphoma and nasopharyngeal carcinoma.

HBV (choice B) is associated with hepatocellular carcinoma.

HHV8 (choice C) is associated with Kaposi's sarcoma.

HTLV-1 (choice E) is associated with adult T-cell leukemia.

A 23-year-old woman with a history of sickle cell disease presents with fever and severe bone pain
localized to
her left tibia. X-ray reveals a lytic lesion and blood cultures reveal infection. A bone culture grows
gram-negative
rods. Which of the following best describes the infecting organism?

A. It is a facultative intracellular parasite

B. It is a nonmotile facultative anaerobe

C. It is comma-shaped and sensitive to acidic pH

D. It is motile and does not ferment lactose

E. It is motile and oxidase positive

Explanation:

The correct answer is D. The presence of sickle cell disease in a question stem is usually a significant
clue.
This question tests if you know that patients with sickle cell anemia are more susceptible to
osteomyelitis
caused by Salmonella. (The patient's fever, bone pain, and x-ray results indicate osteomyelitis). But
note that
Staphylococcus aureus (gram-positive coccus) is the most common cause of osteomyelitis in both
sicklers and
nonsicklers. If it had not been ruled out on bone culture, you should have looked for it in the answer
choices.
Notice that you were required to know more than just the organism's name; you needed to know its
distinguishing features. Choice D describes Salmonella (a gram-negative rod) accurately. Salmonella
exists in
over 1800 serotypes and is known to contaminate poultry.

A facultative intracellular parasite (choice A) is Legionella, a catalase-positive gram-negative rod. It


contaminates air-conditioning cooling towers and causes Legionnaire's disease (a type of pneumonia).

A nonmotile, facultative anaerobe (choice B) is Shigella, a gram-negative rod that does not produce
H2S. All
Shigella contain an endotoxic lipopolysaccharide. The organism causes bacillary dysentery, with
abdominal
cramps, fever, and mucoid, bloody diarrhea.

A comma-shaped organism that is sensitive to acidic pH (choice C) is Vibrio cholerae, a gram-


negative rod that
causes severe enterotoxin-induced diarrhea, with "rice-water" stools and dehydration. The toxin acts
by
stimulating adenylyl cyclase to overproduce cAMP in the brush border of the small intestine.

A motile and oxidase positive organism (choice E) is Pseudomonas, a gram-negative rod with pili that
sometimes produces a polysaccharide slime layer. P. aeruginosa is the prototype and commonly
colonizes the
lungs of patients with cystic fibrosis. It is associated with blue-green pus.

A 42-year-old Hispanic man is brought to the emergency room by ambulance after suffering a grand mal
seizure
at home. There is no history of recent illness, fever, headache, seizures, or head trauma. He drinks
alcohol
occasionally and denies any other drug use. The patient has been a resident of the United States for 15
years,
but occasionally travels to his previous home in Honduras. Neurologic exam shows the patient to be
alert and
oriented. No focal abnormalities are noted. A CT scan of the head reveals multiple punctate
calcifications, and
two enhancing cystic lesions with surrounding edema. What is the most likely diagnosis?

A. Amebiasis

B. Cytomegalovirus infection

C. Echinococcosis

D. Neurocysticercosis

E. Toxoplasmosis

Explanation:

The correct answer is D. Cysticercosis is a parasitic infection caused by the larval cysts of the
tapeworm Taenia
solium. Patients acquire the infection by ingesting the eggs, which reach a larval stage in various
tissues. When
the central nervous system (CNS) is involved, the condition is known as neurocysticercosis. It is the
most
common parasitic infection of the CNS. Infection with this organism is most frequently encountered
in individuals
from Mexico, South Central America, the Philippines, and Southeast Asia. In the CNS, the cysts act
as
space-occupying lesions and can cause hydrocephalus and/or seizures. Seizures are the most common
initial
presentation of patients with neurocysticercosis and may be focal or generalized. Signs of increased
intracranial
pressure such as headache, nausea, vomiting, or visual changes may also be present.

Amebiasis (choice A) is caused by Entamoeba histolytica. Patients typically present with diarrhea
(often bloody),
right lower quadrant abdominal pain, and fever. Amebic abscesses in the liver are a complication due
to
invasion of the portal venous system by the amoeba.

Cytomegalovirus (choice B) produces neonatal infections and infections in immunocompromised


patients such
as AIDS patients. CD4 counts are usually <100 cells/mm3. The most common clinical presentation is
chorioretinitis producing floaters, visual field deficits, and painless loss of vision. CMV also produces
encephalitis and may produce calcifying lesions in the CNS.

Echinococcosis (choice C) is a parasitic nematode infection caused by either Echinococcus granulosis


or E.
multilocularis. The disease is hydatid cyst disease. The patient ingests the eggs from dogs and
becomes an
intermediate host. The cysts are classically in the liver, are calcified, and the patient shows
eosinophilia.

Toxoplasmosis (choice E) is a protozoan infection acquired by eating undercooked meat or by


exposure to cat
feces. Primary toxoplasmosis is usually asymptomatic. In patients with normal immunity, the
organism can cause
a heterophile-negative, mononucleosis-like syndrome. In patients with AIDS, it causes ring-enhanced
focal brain
lesions and pneumonia.

Potrebbero piacerti anche